You are on page 1of 41

Answers

The answers to the practice questions below are as given to the IB examiners. The following notes may help you to
interpret these and make full use of the guidance given.
• There are no half marks awarded. Each mark is shown by the number in brackets (1).
• Points worth single marks are separated from each other by a semicolon (;).
• Alternative possible answers are separated from each other by a dash (/).
• Any answer given in bold or underlined must be present to score the mark.
• Information in brackets ( ) is not needed to score the mark.
• Notes given in italics are to guide the examiner on what to accept/reject in their marking.
• OWTTE means ‘or words to that effect’, so alternative wording that conveys the same meaning can be equally rewarded.
• ECF means ‘error carried forward’, so examiners must award marks for an incorrect answer from an earlier part of a
question used correctly in a subsequent step.
• MP1, MP2 etc. represent method marks to be awarded by an examiner for answers showing the appropriate steps of
the working (method) necessary for answering the question.
You may notice occasional differences between the calculations or wordings given in the answers and those in the
worked solutions. This is because the answers give
the final solution with the minimum of working, and the worked solutions provide the extra reasoning and working
needed to understand how the answers are attained.

13 1600 ± 4 m
Skills 14 1.12 ± 0.01 s
1 (a) 4.8 × 104 (b) 3.6 × 10–5 15 (a) 5.2 cm (b) 4.8 cm
(c) 1.45 × 104 (d) 4.8 × 10–7 (c) 3 cm (d) 8.8 cm
2 (a) 5.59 × 106 m (b) 1.75 m 16 (a) 5 cm (b) 5.66 cm
(c) 2.54 × 10–5 m (d) 1026 m (c) 6.32 cm (d) 3.61 cm
3 (a) 2.68 × 109 s (b) 2.5 × 10–3 s 17 8.94 km, 63.4° north-west
(c) 3.46 × 105 s (d) 1.04 × 104 s 18 112 km, 26.6° north-east
4 –1
(a) 2 × 10 kg –8
(b) 1 × 10 kg 19 8.66 km
(c) 2 × 103 kg 20 7.52 km
5 150 m3 21 433 m
6 (a) 1.0 × 10–10 m3 (b) 1.09 × 1021 m3
7 180 kg
Challenge yourself
8 86.85 kg 1 8 ± 4 m s−2
9 5.48 × 103 kg m–3
10 (a) There is some variation in the mass of the apples,
but the number of apples is proportional to the
A.1
mass of the apples.
(b) There is a larger variation in the mass of the apples. Exercises
(c) The mass of the apples appears to be linearly 1 (a) 27.8 m s–1 (b) 5.6 m s–1
related to the number of apples but there
2 (a) 4.2 m s–1 (b) 0 m s–1
might be a large systematic error in the mass –1
measurement, or the apples were counted (c) –4.2 m s (minus sign indicates traveling in
incorrectly. opposite direction to initial direction)
(d) 90 m
11 A = 4B2
3 22.4 m s–1, 26.6°
12 9056 ± 560 kg m−3

1
v
travelling south)
3 22.4 m s–1, 26.6° v
3 22.4 m s–1 –1, 26.6°
4 4.1 m s–1 , direction is 14° W of S
Answers
s
4 4.1 m s–1 –1 , direction is 14° W of S
s
5 31.6 m s–1
15 velocity = zero
5 31.6 m s–1–1
64 494.1mmss–1–1 , direction is 14° W of S 15
15 displacement velocity = zero
631.649mms–1–1
s–1
–1 A
displacement
75 –30 ms A
6 749 m –30s–1m s–1–1
8 40 m –1 negative gradient C
7 –30 m s
8 40 m negative gradient C
98 3.6
40 mm s–1
9 3.6 m
–1 s–1
–1 B D time
109 43.6
s ms B D time
10104 s 4 s velocity
11 125 m, 2.5 m s–2 velocity
11 125 m, 2.5 m s–2 –2
11 125 m, 2.5 m s–2
1212 15
15 mm
12 15 m A B C D time
1313 A B C D
s, v, a

time
13
s,s,v,v,aa

16 S
s
16
16 S
s constant gradient
v constant gradient
v

a t
a t
t t
The displacement
The displacement graph
graphisisquadratic int the
in the
quadratic firstfirst
and t
last sections,
andThe and linear
lastdisplacement in
sections, andgraph between.
linear is
in quadratic
between. in the first
14
14 and last sections, and linear in between.
s, v, a

17 78 mV
a 17 78
78 m
17 m
18 (a) 30° (b) 17.3 m
18 (a) 30° (b) 17.3 m
18 (a) 30° (b) 17.3 m
5 m5 m
19 19
t 19 5 m 1
20 39.6 m
1
v Challenge yourself
H_SB_IBGLB_9021_ANS.indd 1 16/07/2014 09:38
1 53 m
2_IBPH_SB_IBGLB_9021_ANS.indd
2_IBPH_SB_IBGLB_9021_ANS.indd 1
1 16/07/2014
16/07/2014 09:38
09:38
s
Practice questions
15 velocity = zero 1 (a) ​​​​s​ S​​  = 18 × t​ (1)
displacement (b) 81 m (1)
A (c) 27 m s (1) –1

(d) ​ x = 27 (​ t – 6.0)​​ (1)


negative gradient C (e) ​ t = 9.0​s (2)
2 (a) (i) 4 m (1)
(ii) 2.1 s (3)
B D time
(iii) 15 m (2)
velocity

A B C D time
2
Answers

(b) On the way up, air resistance is in the same 4 (a) 4N down slope (b) 4 N, 37.6° relative to the
direction as weight. On the way down, air dashed line
resistance is in the opposite direction to weight. 5 (a) F1 = 8.49 N F2 = 17.3 N,
(b) 
The magnitude of acceleration is greater on the
F3 = 50 N
way up than down so the time to reach the highest
point is smaller than the time back to the ground. (3) 6 (a) F = T sin 30° (b) 10 = T cos 30°
(c) 11.5 N (d) 5.8 N
3 (a)  (2)
7 (a) F = 50 sin 30° (b) 50 cos 30° = N
(c) F = 25 N, N = 43.3 N
8 (a) 2 T cos 80° = 600 N
(b) T sin 80° = T sin 80°
(c) 1728 N
(b) The boy at the top of the hill will kick the ball 9 –3 N s
with a smaller magnitude of velocity than the 10 –4.02 N s
boy at the bottom. The direction of the shot
from the boy at the top will be at an angle closer 11 6.7 m s–2
to the horizontal; the angle of the shot from the 12 997.5 N
boy at the bottom will be closer to the vertical. (2) 13 (a) 3.3 m s–2 (b) 33 N
4 C(1) 14 2 m s–2
5 A(1) 15 682.5 N
6 (a) 0.19 s (2) 16 (a) 40 N (b) 150 m s–2
(b) Now we will use this time (to reach the net) in
a vertical component calculation to find out 17 (a) force on gas = –force on rocket
the height of the ball when in line with the net. (b) force on water = –force on boat
Listing the variables taking downwards to be (c) force on body = –force on board
positive: ​u = 64 sin 7.0°​ ​a = 9.81​ ​t = 0.19​; (d) water exerts unbalanced force on ball, so ball
​s = ut + ​_12 ​  a ​t​​  2​ exerts force on water; reading increases
= 64 sin 7.0 × 0.19 + ​_12 ​ × 9.81 × 0. ​19​​  2​ 18 (a) 31.4 m (b) 0 m
= 1.66​m; (c) 15.7 s (d) 6.4 × 10–2 Hz
This is vertical distance fallen. The height of the (e) 0.4 rad s–1 (f) 0.8 m s–2
ball is ​2.8 − 1.66 = 1.1​m, which means the ball 19 1389 N
passes over the net (height of 0.91 m). (3)
20 15.8 m s–1
(c) 64.4 m s−1(2)
21 14 m s–1
7 5560 m (2)
l cos θ _l
22 (a) 9 m s–1 (b) 30 N
8 t = ​_
(a) ​ ​= ​ ​​
u cos θ u
(2)
​u​​  ​  sin θ
(b) ​​​l = 4 ​_
2 Challenge yourself
g ​​ (3)
1 72 km h–1, 241 km h–1

A.2 Practice questions


1 (a) the direction of the car is changing; hence the
Exercises velocity of the car is changing
OR
1 (a) 10 N to the right
since the direction of the car is changing; a force
(b) 5.8 N, 31° above the horizontal, to the right must be acting on it, hence it is accelerating (2)
2 (a) 40 N (b) 69 N (b) (i) arrow pointing vertically downwards (1)
3 (a) –74 N to the left
(b) 45 N, 63.4° above the horizontal, to the right

3
Answers

(ii) weight 3 (a) 31.6 kN (3)


Do not penalize candidates if they state ‘gravity’. (b) On sliding, μ​  ​​ d​​ ​FN​  ​​ = 100 sin 18.4°​ kN
normal reaction; (2) Components perpendicular to slope:
Do not penalize candidates if they state ‘push of the ​​FN​  ​= 100 cos 18.4°​kN
track on the marble’. ​​μd​  ​​ ​FN​  ​​ × 100 cos 18.4° = 100 sin 18.4°​
(iii) loss in EP = 0.05 × 10 × (0.8 – 0.35); = ​​μd​  ​= _​100 sin 18.4°
​  = tan 18.4° ⇒ ​μd​  ​​= 0.33​ (3)
100 cos 18.4°
​12 ​  m​v​​  2​​; to give v = 3.0 m s–1
gain in ​​EK​  ​= _
(c) (i) 1.26 m s−2 (2)
OR (ii) 48.9 m s (2) −1

use of v = 2gh to give v = _ 4.0 m s–1 at point B; (d) (i) 20 kN (1)
and then use of ​​v​​  2​ − ​u​​  2​ = √​ 2gh ​ ​with v = 4.0 m s–1 (ii) 597.8 m (2)
and h = 0.35 m; to get u = 3.0 m s–1(3)
Do not penalize the candidate if g = 9.8 m s–2 4 (a) (i) 600 N (1)
recognize
(iv) is that resultant
2
force = mv ; to give correct answer:(ii) 8
v =12.247 kg (2)
used. r
≈ 12 m s–1; [2]
= (0.05 x 9.0) = 2.6 N; m ​v​​  ​
2 (iii) 300 N (2)
(iv) recognize 0.175 that resultant force ​= ​_ r ​​​ Accept calculation of angular velocity
N =( mv – mg; )
2
–1(b) If the rope jerks, the acceleration would be large.
 ​​ 0.05 × 9.0 ​ = 2.0 radians s .
== _ r
– 0.5 = 2.1​ N; = 2.6 N​ This
[Totalmeans the tension would increase and, in
2.6 0.175 [4]
10 marks]
​m​rv​​  ​​  − mg​= 2.6[Total turn, that the horizontal component of tension
2
​N = _ – 0.512=marks]2.1 N 3 (a) The work (4)done per unit mass;
2 (a) ratio between (maximum) friction and in bringing a (small test) mass would increase.
from infi nity to A greater frictional force would
2 (a) ratio between (maximum) friction and normal be needed to
normal reaction / OWTTE; [1] the point; [2] remain stationary. (2)
reaction / OWTTE(1)
Don’t accept equation without definitions Idea of ratio crucial for fi rst mark.
5 (a) (i) 4.5 m s (1) −1
Don’t accept equation without definitions of symbols.
of symbols.
(b) (i) g = GM – GM ;
p m

(b) (b)(i)(i) static;


static; (Award(Award thisthis
markmark
for baldfor bald statement even 1
2
2
2
r r (ii) 4.5 m s−1(1)
M M
ifstatement
the reason even isifwrong.)
the reason
sinceis wrong.)
person is not
p
2
0= m
2
0.8

0.2
; (iii) 31.8 N (3)
since person is not moving vertically /
moving vertically / OWTTE;(2) p M = 16; (b) (i) The horizontal [3] component of velocity
OWTTE; [2] M
(ii)(ii) Award [1] for each force labeled to show
m

(ii) KE = mΔV;
changes, so there is an unbalanced force on
Award [1] for each force labelled to
understanding.
show understanding. [3] KE(3)
= 1500 × (4.6 – 0.20) × 10 the
7
; sand. According to Newton’s third law,
friction KE = 6.6 × 1010 J; an equal [3] and opposite force is exerted on
Award [2] if attempted use ofthe ΔV truck. In turn, there must be a frictional
but value used is wrong and force [1] if anopposing this in order for the truck to
individual potential value rather than a
difference is used.)
remain stationary. (3)
(normal) reaction (ii)[Total
22.5 N
8 marks] (2)
4 (a) (i) 6 (± 0.3)
Vsurface = –6.3 C(1)× 107 J kg–1 [1]
(ii) Vh is at R =742 × D(1)
106 m;
floor = –1.0 (±0.2) × 107 J kg–1; [2]
weight Watch for R8= 3.6D(1)
× 107 m being used.
If so award [1] and use ECF.
Use
Use benefit
benefit ofofthe
thedoubt
doubtandand accept, e.g, mg or W for
accept 9 (a) 0 N (1)
things like mg or W for weight etc. (b) ΔV = 5.3 (± 0.5) × 107 J kg–1;
weight etc. Energy = mΔV;
(b) The blades exert a downward force on the air.
Note: ‘centripetal force’ is not a correct
Note: ‘centripetal
label for the reactionforce’
force.is not a[2]correct label for=the
Award 5.3 (±0.5) × 1011 J; The air exerts [3] an equal and opposite force on the
reaction
for correctforce. with no[2]
forcesAward for correct forces withAward
labels. blades
no [2] if student calculates toofform
the PE the a Newton’s third law pair. (2)
(c) (i) labels.
friction, F = mg = 800 N; satellite (1011 J). (c) 8.1 m s−1(3)
R = F or R F 800 = 2000 N; (c) Any two of the following ([1] each): −2
(c) (i) friction, μ
== mg = 800 N;
0.4
[2] (d) 4.6 m s (2)
mv the satellite has to be given a horizontal
​Fμ​  ORuseRof= ​_
2
800= answer to c (i)
​R = _
(ii) attempted
r ​ = 2000 N​
0.4
velocity(2)(or has10 (a)KE) to go into orbit;
to have FF  (2)
i.e. 2000;
rocket motors lifting rocket not 100%
​​m​rvor​​  ​​​equivalent.
2
of _
mv = 800
2
(ii) attempted
Award [0] for use
r
= answer to (c) (i) i.e.effi2000
cient;
FN
_ m​v​​  ​
Note: Watch for ECF. 2 air resistance in initial stages of launch; [2]
Award [0] for ​ r ​​ = 800 or equivalent.
Recall of F = mv not sufficient without
2
[Total 8 marks]
Note: Watch
link to c (i).
r for ECF.

​=m​r150
v​​  ​
2
Recall of80x​F6.0= _
v2 = 2000 ​​ not sufficient without link to (c) (i). W
​200080× 6.0 ​= 150​; to give correct answer:
​​v​​  2​= _ (b) Horizontally, ​​FN​  ​​ = ​mRω​​ 2​​. Vertically, ​​Ff​  ​​ = mg​
2 v =12.247 ≈ 12 m s–1(2) ​​F​ f​​= μ ​FN​  ​​= μ ​mRω​​  2​​
Accept calculation of angular velocity = ​μ ​mRω​​  2
​ = mg​
_


2.0 radians s–1.
M04_IBPH_SB_IBGLB_9021_ANS.indd 2 g 22/07/2014 16:25
ω = ​_
​ ​  μR ​  ​​ (2)
4
Answers

(c) Using (b), the minimum required angular 15 800 W


_
velocity ​= ​ _
0.4 × 3.5 √
​  9.81  ​  ​ = 2.6​rad s−1
28
16 1000 W
17 20 kW
The actual angular velocity ​= 2πf = 2π × ​_
60

= 2.9​rad s −1 18 50%
The actual angular velocity is greater than the 19 42 kJ
minimum, so the person does not slide. (3) 20 (a) 6.67 kW (b) 11.1 kW
11 C(1)
12 D(1)
Challenge yourself
____________
1
√ ( ​m​  ​​  − ​m​  ​​  ) g 1.7 m s–1
v = ​ 2 ​ _
13 (a) ​ ​​ h ​​
​m​  ​​  + ​m​   ​ 
2 1
(2)
2 1
( ​m2​  ​​  − ​m​ 1​​  ) g Practice questions
2 ​ _ ​m2​  ​​  + ​m​ 1 ​ ​​ h ( ​m​  ​​  − ​m​  ​​  )
s=​
(b) ​ _ ​= _
​ ​m​  2​​  + ​m​  1​​ ​  h​ (2) 1 (a) (i) 440 N s (2)
2g 2 1
(ii) −440 N s (1)
(iii) 1740 J (1)
A.3 (b) To reach the other side, the initial kinetic energy
40 97.7 m must be equal to or greater than the change in
potential energy. ​Δ ​Ep​  ​​ = mgΔh
41Exercises
(a) –1300 J (b) dog = 180 × 10 × 1 = 1800​J
1 (a) –1300 J (b) dog The kinetic energy is slightly less than this, so the
42 0 J
2 0J competitor will not reach the far end of the wire. (2)
43 –300 J 2 (a) (i) 320 J (1)
3 –300 J
(ii) 137 J (1)
444 (a) 2 cm
cm (b)(b)4 N4 N
(iii) The masses also gain kinetic energy and,
(c)
(c) F due to conservation of energy, this subtracts
8N from the total energy input.
(b) 34% (2)
3 (a) 14 m s−1(2)
4N
(b) 2.3 m s−1(2)
(c) 794 N m−1(2)
4 B(1)
2 cm 4 cm x
5 A(1)
(d) 44 JJ
(d) (e)(e)1212
J J
6 C(1)
455 1950 JJ
1950 7 C(1)
6 32 m s–1
46 32 m s–1 8 D(1)
7 (a) 12.5 J (b) 12.5 J
9 D(1)
47 (c) 12.5
(a) 1 cmJ (b) 12.5 J
10 C(1)
8 (a) 10.75
(c) cmJ (b) 0.75 m
​  1 ​  m ​v​​  2​
_
489 (a) 0.2 mJ (b)(b)0.75
0.8mm 11 (a) Using conservation of energy, ​​_
2
(a) 0.75 mgh
​= 0.24​
10 (a) 1.96 kJ (b) 4.36 kJ v​ = 11.9​m s–1(2)
49 (a) 0.2 m 5 (b) 0.8 m
11 (a) 3.86 × 10 J (b) 6.43 × 105 J Award GPE lost = 65 × 9.81 × 30 = 19 130 J
50 (c)
(a) 10–2 l
1.8 ×kJ
1.96 (b) 4.36 kJ Must see the 11.9 value for MP2, not simply 12.
12 (a) 1.25 × 10–45 J (b) 6.25 × 10–5 J Allow g = 9.8 ms−2.
51 (a) 3.86 × –1 10 J (b) 6.43 × 105 J
(c) 0.1 m s
(c) 1.8 × 10–2 l
13 (a) –10.83 m s –1 (b) 521.3 J
5214 (a) 1.25 ×
velocities 10–4 J
swap (b) 6.25 × 10–5 J
(c) 0.1 m s –1
5
53 (a) –10.83 m s –1 (b) 521.3 J
Answers

(b)  (2) (ii) force acts on block OR gravity/component


Ff
of weight pulls down slope;
velocity/speed decreases OR it is slowing
down OR it decelerates (2)
(c) straight line through origin for at least one-third
of the total length of time axis;
W
followed by curve with decreasing positive
arrow vertically downwards from dot labeled gradient
weight/​W​/​mg​/gravitational force/​​Fg​  ​​​ AND arrow  (2)

displacement
vertically upwards from dot labeled reaction
force/​R​/normal contact force/​N​/​​FN​  ​​​;
​W > R​ (2)
Do not allow gravity.
Do not award MP1 if additional ‘centripetal’ force arrow 0
0 time
is added.
Arrows must connect to dot. (d) 640 N (2)
Ignore any horizontal arrow labeled friction. (e) 0.55 OR 0.547 OR 55% OR 54.7% (2)
Judge by eye for MP2. Arrows do not have to be correctly 13 (a) C (1)
labeled or connect to dot for MP2 (b) Although equal and opposite forces act between
(c) For the skier to remain in contact with the the car and the magnet, they are connected and
circular slope, a centripetal force is required cancel. Alternatively, no overall work is done
(allow centripetal acceleration). and therefore no kinetic energy is produced. (1)
​​ m ​rv​​  ​​= _
​65 20
× ​12​​  2​
2
_ ​ = 468 ​N 14 (a) ​​_23 ​​ (2)
The centripetal force is weight − normal (b) 89% (2)
reaction force​.​Since the weight is larger than the
(c) 10 (2)
centripetal force, he skier does not lose contact
with the ground. (3)
There are several alternative ways of reaching the
conclusion, but do not award a mark for the bald A.4
statement that the skier does not lose contact with the
ground. Exercises
(d) 0.14 (3)
1 300 g
(e) 730 N s (2)
(f ) The change in momentum is the same with or 2 0.2 m
without a flexible net, because the skier goes 3 (a) 10 000 N
from having a speed to having zero speed. The (b) 267 N
net increases the stopping time. (c) 3000 N
Δp
​ = ​_ ​​, ​F​is smaller, which means that the
Since F 4 210 N at one end and 690 N at the other end
Δt
body experiences less force (and harm).
5 0.1 m
(2)
6 (a) 509 N
12 (a) (i) Using conservation of energy between the
(b) 360 N
compressed spring and the top of the ramp, ​​
(c) 240 N
​12 ​  m​v​​  2​  + mgh​
E​ p, elastic​= _
7 (a) 933 N
​12 ​ × 55 × 0. ​90​​  2​ + 55 × 9.81 × 1.2
​​Ep,​  elastic​= _
(b) 660 N
= 669​J (2) (c) –60 N
Award [1 max.] for use of g=10 N kg , −1
8 75 N
gives 682 J.
9 0.375
(ii) 4.9 m s−1(2)
(b) (i) no force/friction on the block, hence 10 (a) 16 rad s–1
constant motion/velocity/speed. (1) (b) 8.75 revolutions

6
Answers

11 (a) –25π = –78.54 rad s–2 Practice questions


(b) 0.4 s
1 (a) 245 kg m2(3)
12 (a) 0.4 rad s–2
(b) (i) 300 N m (1)
(b) 1 m s–2
(ii) 2000 J (1)
13 (a) 0.5π = 1.57 rad s–1 (iii) 2000 J (1)
(b) child at 2 m: π = 3.14 m s–1; (iv) 4.0 rad s−1(1)
child at 0.5 m: 0.25π = 0.79 m s–1 (v) 980 kg m2 s−1(1)
(c) child at 2 m: 98.7 N, child at 0.5 m: 24.7 N (c) (i) the child accelerates, which means there is
14 32 rad s–2 an unbalanced force; this is supplied by the
roundabout; according to Newton’s third
15 7.85 N
law, the child exerts an equal and opposite
16 15 rad s–2 force on the roundabout, so the roundabout
17 15 rad s–2 slows down (3)
An answer in terms of energy is also acceptable,
18 (a) 500 rad s–2
e.g. the child gains rotational kinetic energy, which
(b) 7.96 revolutions
subtracts from the roundabout’s rotational kinetic
(c) 224 rad s–1 energy due to conservation of energy.
19 (a) –0.1 N m (anticlockwise) (ii) 2.4 s−1(2)
(b) –250 rad s–2 (iii) 461 N (2)
(c) 2.51 s (d) (i) 0.98 rad s−2(2)
20 (a) 500 N m (ii) invalid because angular acceleration is not
(b) 3 rad s–2 constant; the answer obtained for time
would be lower than in reality (2)
21 (a) 79 J
(b) 316 J 2 (a) 10 N (2)
(c) 0.012 J (b) (i) There must be a horizontal frictional force
at the bottom. (2)
22 8 J ___
(ii) 15 N. (3)

6gh
23 v = ​​ ___
​  5 ​
 ​​the solid ball takes less time
3 (a) Using ​​​ωf​  ​​​​  2​ = ​​ωi​  ​​​​  2​ + 2αΔθ​,
​​​ωf​  ​​​​  2​= 0 + 2 × 0.110 × 6 × 2π​
24 (a) 0.25 J ​​ωf​  ​= 2.88​rad s−1(2)
(b) 0.85 m s–1 At least 3 s.f.
(c) 0.29 m Other methods are possible.
(d) 0.68 s Answer 3 given so look for correct working.
25 3.14 × 10–2 kg m2 s –1 (b)
θ/rad

26 0.157 kg m2 s –1
27 (a) 4.675 kg m2
(b) 0.925 kg m2
(c) 5.1 revolution s–1 0
0 t/s
(d) before: 92.3 J; after: 475 J
concave up from origin (1)
(e) work done pulling her arms in
(c) 2.38 × 10−3 N m (1)
28 (a) 1.125 × 10–2 kg m2 (d) 130 s (2)
(b) 1.225 × 10–2 kg m2 Various methods are possible.
(c) 0.92 π = 2.9 rad s–1 Award [2] marks for a bald correct answer.
29 (a) 0.5 kg m2 s –1 4 (a) the person rotates anticlockwise;
(b) 5 m s–1 this is because of conservation of angular
momentum; in order for the wheel to rotate
clockwise, the person’s angular momentum must
be opposite to this (3)
7
Answers

(b) the rotational kinetic energy has increased;


because the person has done work when flipping A.5
the wheel (2)
5 Using conservation of rotational and linear energy: Exercises
​mgh = ​ ​  m​v​​  ​ + ​_12 ​  I ​ω​​  2​​
_1
2
2
1 (a) 8.5 m s–1
Now, ​I = _ ​25 ​  m​r​​  2​​ and ​ω = _​vr ​​ (b) 10 m
10gh (c) 170 m
Correct manipulation to show that ​​v​​  2​= _
​ 7 ​​ (3)
2 (a) 1.34
6 (a) A torque requires a force and for there to be
(b) t′ = –2.44 × 10–7 s, x′ = 123.4 m
a distance between the force and the axis of
rotation.(1) 3 event 1 at 6.68 × 10–6 s, event 2 at 6.22 × 10–6 s
(b) (i) Considering translational motion and 4 2.8 s
components along the slope:
5 213 s
​Mg sin θ − F = Ma​
Considering rotational motion: 6 (a) rocket observer uses same clock so measures
proper time = 2 years
​​Ff​  ​​  R = Iα​
(b) 3.3 years
Iα Iα
Substituting ​​Ff​  ​=  ​_
R
​​ gives ​Mg sin θ = Ma + ​_
R
​​
7 1.43 m
​Ra ​​,
Using ​I = ​_12 ​  M​R​​  2​​ and ​α = _ 8 (a) 5.94 m
a
​  12 ​  M​R​​  2​ ​ _
_
R
​ (b) 1.42 × 10–7 s
​Mg sin θ = Ma + ​ R _ ​​
(c) 42.2 m
​32 ​  a​
​g sin θ = a + ​_12 ​  a = _ (d) proper time is measured by nucleus frame of
​a = ​_23 ​  g sin θ​ (4) reference
(ii) 0.96 s (2) (e) proper length measured by Earth observer
(c) The ice slides whereas the cylinder rotates. 9 (a) 6.25 hours
In terms of energies, the cylinder’s potential (b) 3 light hours
becomes both kinetic and rotational kinetic. (c) 3.75 hours
In terms of accelerations, the cylinder’s is
10 –0.99 c
always less than that of the ice. Through either
consideration we can see that the speed of the 11 0.85 c
cylinder will be less at any given point. (1) 12 0.96 c
Allow answers in terms of energies, e.g. ice does not 13 –9.86 × 103 m, –9.87 × 103 m
use energy to rotate and therefore will have a greater
translational speed. 14 from graph: t′ = 5 years, x′ = 1 light year
(d) the hollow cylinder has a greater moment of from Lorentz transformation: t′ = 5.2 years,
inertia; so the acceleration will be smaller for the x′ = 0.86 light years
same torque (2) S = –26.3 ly2
7 (a) B (1) S′ = –24 ly2
(b) The road exerts a forward force on the tires. 15 Both graph and Lorentz transformations agree, to give:
The line of action of the force is along the t = 4 years, x = 6.3 light years
roadway, which means the torque sees the front S = –23.7 ly2
of the car rotate upward. (1) S′ = –24 ly2
8 (a) D (1) 16 4 years
(b) Linear momentum is always changing because
17 2.6 light years
the speed of the swing is always changing.
Angular momentum is always changing because 18 7 years
the angular speed of the swing is always changing.(1) 19 Depart 2003, arrive 2000; if 2c then simultaneous,
can’t calculate y if v > c

8
Answers

Practice questions 4 (a) (i) The c​ t'​axis is the same as a body traveling at
the same velocity as the observer.
1 (a) proper time: the time interval measured by an ​tan θ = _​vc ​= 0.5​
observer of an event that happens at the same
​v = 0.5c​ (2)
place according to that observer;
(ii) 1.16 (2)
proper length: the length of an object as measured
by an observer who is at rest relative to the object(2) (iii) ​t = 4​s and ​x = 3​light seconds (1)
Do not look for precise wording but look for the (iv) ​​t′ ​= 3​s and ​​x′ ​= 1​light second (1)
understanding of the quantities in the sense of the words. (v) ​​x′ ​= γ​(x − vt)​= 1.16​(3c − 0.5c × 4)​
(b) (i) no they will not appear to be simultaneous; = 1.16​light seconds ​≈ 1​light second
​t' = γ​(t − ​ _2 ​)​= 1.16​(4 −  ​  )​
vx _ 0.5c × 3c
Look for a discussion along the following lines. 2 ​
​c​​  ​ ​c​​  ​
Carmen sees Miguel move away from the = 2.9 s ≈ 3 s​ (2)
signal from A and since Miguel receives the two
(b) (i) 4c(1)
signals at the same time; and since the speed of
(ii) It would arrive before it left (2)
light is independent of the motion of the source;
(c) (i) 4 s (1)
Carmen will see the light from A first / light
from B will reach Carmen after light from A / (ii) 4.6 s (1)
OWTTE(4) (iii) 4 s because the start and end are measured
(ii) γ = 2; to give u = 0.87 c (2.6 × 108 m s–1)(2) by same clock on spaceship. (1)
(iii) both measure the correct distance; SR states (iv) Time intervals are longer;
that there is no preferred reference system / when measured by observers moving
laws of physics are the same for all inertial relative to events. (2)
observers / OWTTE(2) 5 It is possible to respond to this question in terms of
2 (a) frame moving with constant velocity / frame in time or distance.
2000
which Newton’s first law is valid (1) Earth frame transit time ​= ​_
0.98c
​​ = 6.8 μs
​2D
(b) ​​T​ 0​= _ c ​​ (1) Earth frame dilation of proper half-life
(c) (i) light reflected off mirror when midway = 2.2 ×
​ ​5 = 11 μs
6.8
between F and R(1) Muon’s proper transit time ​= ​_
5
​​= 1.4 μs
(ii) F – R = vT(1) Distance muons can travel in a proper lifetime
(iii) ​​​(__
​  12 ​  L)​​​  ​= ​D​​  2​ + ​( ​  12 ​  vT)​​​  ​​
2 2
​ __ = 2.2 ×
​ ​0.98c = 650 m
______________ Earth frame lifetime distance due to time dilation
​L = 2 ​ ​(
√ ​  12 ​  vT)​​​  ​)​  ​​
​D​​  2​  + ​​(__
2
   (2) = 650 ​×​5 = 3250 m
2000
______________ Muon frame distance traveled ​= ​ _5
 ​​= 400 m (3)


2 ​ ​( ​  12 ​  vT)​​​  ​)​ ​
​D​​  ​  + ​​(__
2
   2
Accept answers from one of the alternatives.
(iv) ​​T​ 0​=   
​ _____________
​​;
c 6 E(1)
​​c​​  2​ ​​T0​  ​​​​  2​= 4​(​D​​  2​  + ​​(__
​  12 ​  vT)​​​  ​)​​;
2

7 the person will experience their own mass as though it


use od 4​  ​D​​  2​= ​c​​  2​ ​​T0​  ​​​​  2​​; is normal, irrespective of their speed; they can increase
​T​  ​​ their speed as much as they want, but the resulting
hence ​T = _
​_
0
​​ (4) speed is not the sum of the additions (2)

​ ​(1 − ​ _ 2 ​)​ ​
​v​​  ​
2

​c​​  ​
8 (a) D (1)
3 (a) the speed of light in a vacuum is the same for all
(b) It depends on your frame of reference (with the
inertial observers; the laws of physics are the same
pipe, with the spear, or halfway between the two
in all inertial frames of reference (2)
speeds).(2)
(b) (i) this faster than light speed is not the speed
of any physical object / inertial observer and
so is not in violation of the theory of SR (1)
​u −_
u' = _
(ii) ​ v
uv ​​ with ​v = − 0.80c​and ​u = 0.80c​
1 = ​  2 ​
​c​​  ​
​ 0.80c
   + 0.80c
so that ​u’ = ____________
   0.80c × 0.80c
___________
​​ ;
1 +   ​   ​
​c​​  2​
_ 1.60c
u​ ' = ​1.64 ​ = 0.98c​ (3)

9
Answers

(ii) If no water is present then the heat loss from


B.1 the element will be reduced.
The element will reach a higher temperature. (2)
Exercises 2 (a) 2773 K (4 s.f.) (2)
1 (a) 3.92 × 10 J 3
(b) 3.92 × 10 J3 (b) conduction and convection require particles;
radiation is the only mechanism of heat loss in a
2 6
1.8 × 10 J vacuum(2)
3 3.7 × 104 J (c) ​1.4 × ​10​​  −4​​ m (3)
4 67.6 J, 13.5 N 3 (a) (165, 0) (1)
5 10 °C (b) Look for these points:
6 (a) 6 × 10–21 J (b) 4.8 × 10–26 kg to change phase, the separation of the molecules
must increase;
(c) 500 m s–1
Some recognition that the ice is changing phase is needed.
7 (a) 5.8 × 10–6 m
so all the energy input goes to increasing the Ep of
(b) 3.54 × 103 W m–2 the molecules;
(c) 18 W Accept something like ‘breaking the molecular bonds’.
(d) 1.4 W m–2 Ek of the molecules remains constant, hence
8 420 kJ temperature remains constant (3)
9 (a) 3.6 × 106 J (b) 3.6 × 105 J °C–1 If Ek mentioned but not temperature then assume
(c) Some heat is lost to the outside. students know that temperature is a measure of Ek.
(c) (i) time for water to go from 0 to 15°C = 30 s;
10 1.33 × 104 J
energy required = ​mcΔθ​= 0.25 × 15 × 4 200 =
11 (a) 1 kg (b) 3.36 × 105 J 15 750 J;
(c) 336 s energy
​power = _
​ time ​= 525 W ≈ 530 W​ (3)
12 (a) 3 × 105 J (b) 686.7 °C (ii) ice takes 15 s to go from –15°C to 0;
13 (a) 3 × 105 J (b) 2.25 × 105 J energy supplied = 15 × 530 J;
​(530 × 15)​
(c) 51 °C ​specific heat = _
​​(15 × 0.25)​​ = 2100 J ​kg​​  −1​ ​K​​  −1​​ (3)
14 (a) 80 kg (b) 1.34 × 107 J (iii) time to melt ice = 150 s;
​​ 1500.25
× 530 ​
15 3.35 × 1011 J ( )
​L = _ ​ = 320 kJ ​kg​​  −1​​ (2)
16 1.135 × 10 s 3
4 (a) [1] for each appropriate and valid point e.g. thermal energy
17 (a) 1.84 × 104 kg (b) 6.16 × 109 J is the Ek of the component particles of an object; thus
(c) 3.42 × 105 W (d) 342 W m–2 measured in joules; the temperature of an object is a
measure of how hot something is (it can be used to
Practice questions work out the direction of the natural flow of thermal
energy between two objects in thermal contact) /
1 (a) (i) work is done against friction; which means
measure of the average Ek of molecules; it is
that energy is transferred to the atoms in
measured on a defined scale (Celsius, kelvin, etc.) (4)
the bar; temperature is linked to the average
(b) (i) correct substitution: energy = power × time;
kinetic energy of the atoms (3)
= 1200 W × (30 × 60) s;
(ii) 0.063 kg (2)
= 2.2 × 106 J (2)
(iii) 8.7 N (3)
(ii) use of ​Q = mcΔθ​;
(b) (i) 31 s (2)
​ 2.2 × ​10​​  ​) ​ = 7.5 K​
6

(ii) 1000 J (1) ​ θ =_


to get Δ (3)
( ​ 4200 × 70 ​
(c) (i) convection currents carry hot, less dense fluid (c) [1] naming each process up to [3].
upwards; if the element were at the top, only convection; conduction; radiation;
very small currents at the top could form; [1] for an appropriate (matching) piece of information /
the boiler does not need to be filled to the top outline for each process up to [3].
because water will leave from near the bottom (2)

10
Answers

e.g. convection is the transfer of thermal energy Challenge yourself


via bulk movement of a gas due to a change
of density; conduction is transfer of thermal 1 T = 276 K
energy a intermolecular collisions; radiation is
the transfer of thermal energy via electromagnetic Practice questions
waves; (IR part of the electromagnetic spectrum 1 (a) (natural process of ) production takes thousands /
in this situation) / OWTTE(6) millions of years; fossil fuels used much faster
(d) (i) [1] for each valid and relevant point e.g. in than being produced / OWTTE(2)
evaporation the faster moving molecules (b) Any two sensible suggestions e.g. storage
escape; this means the average Ek of the of radioactive waste; increased cost; risk of
sample left has fallen; a fall in average Ek is radioactive contamination etc. (2)
the same as a fall in temperature (3) To achieve full marks the differences must be distinct.
(ii) energy lost by evaporation
2 (a) solar panel: solar energy ➞ thermal energy (heat);
= 50% × 2.2 × 106 J;
solar cell: solar energy ➞ electrical energy; (2)
= 1.1 × 106 J;
(b) (i) input power required = 720 W (± 5 W);
correct substitution into E = ml
720
1.1 × 106 J    
to give mass lost ​=  ​ ____________
   ​area = ​_ ​= 0.90 ​m​​  2​​; (2)
−1 ​​ 800
2.26 × ​10​​  ​  J ​kg​​  ​
6
(ii) power extracted ≈ 150 W (± 20 W);
= 0.487 kg = 487 g (3)
power out 150
(iii) [1] for any valid and relevant factors up to [2] e.g. e​ fficiency = _
​power in ​  OR  ​_
500
​​ = 30%; (allow ECF)
area of skin exposed; presence or absence of (3)
wind; temperature of air; humidity of air etc.; 3 (a) the solar radiation is captured by a disk of area
[1] for appropriate and matching explanations up πR2 where R is the radius of the Earth; but is
to [2] e.g. increased area means greater total distributed (when averaged) over the entire Earth’s
evaporation rate; presence of wind means surface, which has an area four times as large; (2)
greater total evaporation rate; evaporation rate Award [1] for reference to absorption/reflection.
depends on temperature difference; increased (b) (i) 0.700; (1)
humidity decreases total evaporation rate (4)
(ii) ​ I(​ = eσ ​​TA​  ​​​​  4)​ ​​;
5 D(1) ​= 0.700 × 5.67 × ​10​​  −8​ × ​242​​  4​ = 136 W ​m​​  −2​​ (1)
6 A(1) (iii) ​σ ​​TE​  ​​​​  4​ = 136 + 245 W ​m​​  −2​​
_
hence ​​TE​  ​= ( ​  381 −8 ​ ​)​ = 286 K​
7 A(1)

4
​ = ​ _ (2)
5.67 × ​10​​  ​
8 B(1)
(c) (i) the Earth emits radiation in the infrared region of
9 ​ L 2 ​​
(a) Substitution of ​L = σA ​T​​  4​​ into ​b = _ the spectrum; the greenhouse gases have energy
4π ​d​​  ​
σA ​T​​  ​
4 level differences (in their molecular energy levels)
​b = ​_ 2 ​​ and ​σ​ and ​4π​are constants. (1)
4π ​d​​  ​ corresponding to infrared energies; and so the
(b) Sirius is a star and is therefore luminous. infrared photons are absorbed
Venus is a planet; it is bright in the night sky OR
because it reflects the Sun’s light. (2) the Earth radiates photons of infrared
10 D(1) frequency; the greenhouse gas molecules
oscillate / vibrate with frequencies in the
11 Cu/Al junction: ​17.9° C​; Fe/Cu junction: ​28.6° C.​ (5)
infrared region; and so because of resonance
the photons are absorbed (3)
(ii) most incoming radiation consists of photons
B.2 in the visible / ultraviolet region /
photons of much shorter wavelength than
Exercises those radiated by the Earth / photons of
different wavelength from that radiated by Earth;
1 (a) 531 W m–2 and so these cannot be absorbed (2)
(b) 31 W m–2 (iii) Source: emissions from volcanoes / burning
(c) 0.31 of fossil fuels in power plants / cars / breathing;
(d) 531 W m–2 Sink: oceans / rivers / lakes / seas / trees (2)
(e) 10.3%
11
Answers

4 (a) energy emitted per unit time / power per unit 10 D(1)
area; proportional to [absolute temperature / 11 D(1)
temperature in K]4(2)
Must define symbols if used. 12 A(1)
(b) (i) ​power 13 D(1)
= 5.67 × ​10​​  –8​ × 4π × ​​[7.0 × ​10​​  8]​ ​​​  2​  × ​5800​​  4​​;
​≈ 4.0 × ​10​​  26​  W​ (1)
incident energy
(ii) ​​___________
area
3.96 × ​10​​  26​
​ = ​____________
[ 11] 2 ;​​
4π ​​ 1.5 × ​10​​  ​ ​​​  ​
B.3
= 1400 W m−2(2)
(iii) two of: Exercises
(albedo of Earth means) some radiation is 1 (a) 7.12 × 10–6 m3
reflected; Earth’s surface is not always normal (b) 6.022 × 1023 atoms
to incident radiation; some energy lost as (c) 1.2 × 10–29 m3
radiation travels to Earth (2)
2 27 g
(iv) power absorbed = power radiated; uses
_
3 292 kPa

4
​  240
5.67 × 10–8 × 2554 = 240 / evaluates ​​ _ σ ​  ​​ (2)
4 (a) 6 kPa (b) 3 kPa
(c) radiation from the Sun is re-emitted at longer
wavelengths; (longer radiation) wavelengths are 5 312.5 cm3
absorbed by greenhouse gases; some radiation 6 400 kPa
re-emitted back to Earth (3)
7 9.4 kJ
(d) more CO2 / named greenhouse gas released
into atmosphere; enhanced greenhouse effect; 8 8.28 × 10–21 J
because more reradiation of energy toward
surface(3) Challenge yourself
5 (a) (i) the black pattern will be a better radiator; 1 P = 114.3 kPa
because the emissivity is higher (2) 1
_​  ρ ​v​​  2​​and the ideal gas
2 We know that pressure, ​P = ​ 3
(ii) black surface (1) law states ​PV = NkT​.
(iii) white surface (1) Substituting pressure into the ideal gas law gives
(b) white roofs would reflect more radiation; 1
​​_3 ​  ρ ​v​​  2​  V = NkT​. Since the mass of all molecules is

the temperature of the Earth would not have to ρV 1
be so great; to achieve equilibrium with power in ​ρV​, the mass of one molecule, ​m = ​_ _
N ​​, so ​​​  3 ​  m ​v​​  ​​  = kT​
2

equal to power out (3)


(c) (i) all infrared radiation is already absorbed on Making kinetic energy the subject of the equation:
entry; which means increased proportions _ ‾
​​​Ek​  ​​​ = ​_
1 3
​  2 ​  m ​v​​  2​​ = ​_2 ​  kT​
of greenhouse gases could not absorb and
re-emit any more (2)
(ii) there is no UV filter; UV radiation is harmful Practice questions
to DNA (2)
(iii) clouds reflect some wavelengths of radiation; 1 (a) (i) 0.83 m3(2)
which increases the albedo (2) ​m
(ii) 1.06 m3; ​ρ = _ 1
​= ​_
V 1.566
​ = 0.95​kg m−3(3)
6 (a) (i) 228 W (2) (iii) 2380 kg (1)
(ii) −2
4.5 W m (2) (iv) 3030 kg (1)
(b) (i) 0.14 J (2) (v) Upthrust (30 300 N) > weight (23 800 N) so
(ii) all radiation is absorbed; the sensor area is the balloon will rise. (2)
perpendicular to the radius (2) (b) (i) 570 m s−1(2)
7 A(1) (ii) ​7.96 × ​10​​  4​​ moles (2)
(iii) ​3.45 × ​10​​  8​​ J (2)
8 B(1)
9 C(1)

12
Answers

1
2 (a) (i) random motion; no gravitational effect; (e) Very large _
​​  H ​​means very small volumes / very
no forces of attraction between molecules/ high pressures; at very small volumes the ideal
atoms; time of collision much less than time gas law does not apply (2)
between collisions; Newton’s laws apply (2)
4 A(1)
(ii) potential energy not used/irrelevant;
(because) there are no forces between 5 D(1)
molecules in an ideal gas; gas speeds vary so 6 B(1)
need to take an average (3)
7 B(1)
(b) (i) ​n=_​PV
RT
​;= 0.18 mol​ (2)
8 B(1)
Award [2] for bald correct answer.
9 C(1)
(ii) show use of PV = constant; 1.9 × 106 Pa (2)
Award [2] for bald correct answer. 10 B(1)
​420 ×290
19 × ​10​​  ​
(iii) ​pressure = ___________
5
​​ 11 (a) 48 kPa (1)
(b) 0.012 kg (1)
= 2.8 × 106 Pa (1)
(c) ​6.5 × ​10​​  ​  J​
−21
(1)
OR
​pressure = ​(_
​  V ​=)​ ​______________
nRT 0.18 × 8.31 × 420 (d) larger temperature implies larger (average)
   2.3 × ​10​​  −4​
​​
speed/larger (average) kinetic energy of
= 2.8 × 106 Pa molecules/particles/atoms;
(c)
(c) increased force/momentum transferred to walls
P
(per collision) / more frequent collisions with walls;
increased force leads to increased pressure
(b)(iii) because P=F/A (as area remains constant) (3)
(c) Ignore any mention of PV = nRT.

(b)(ii)
A B.4
V Exercises
smooth
smoothcurve, curving
curve, correct
curving way for
correct way(b)for
(ii); 1 (a) 700 K
vertical straight line for (b) (iii); smooth curve,
b (ii); (b) 350 K
steeper than (b) (ii) for (c) (3)
vertical straight line for b (iii); (c) 5.25 J
Labeled curves are not needed as such but direction must
smooth curve, steeper than b (ii) for c; (d) 3.5 J
be clear.
[3] (e) heat loss = 8.75 J
3 (a) in order to keep the temperature constant OR
Labelled
in order curves
to allow are nottoneeded
the system as
reach thermal 2 (a) 1050 K
such but direction must
equilibrium with the surroundings. be clear. (1) (b) 11.6 J
Accept answers in terms of pressure[Totalor volume changes
13 marks] (c) 11.6 J
__
5
only if clearly related to reaching thermal equilibrium with 3 (a) ​​PV ​ ​​ 3  ​ ​​ = constant
the surroundings. (b) 8.3 J
(b) 0.5 Pa m (3) (c) 1040 K
Challenge
recognizes yourself
b as gradient; calculates b in range (d) 510 K
4.7 × 104 to 5.3 × 104 (e) –7.95 J
Award [2 max] if POT error in b. (f) 0.35 J (should be 0 J)
1 P = 114.3 kPacorrect SI unit, e.g. kg​ ​s−2
Allow any
1
​ ∝_
(c) ​V ∝ H​so p ​H ​​.
so graph should be a straight line through origin,
as observed (2)
(d) 0.03 (2)
Answer must be to 1 or 2 s.f.

13
(c) 1040 K right and up the page; 44 67 cm3
associated with area within the rectangle;
(d) 510 K correctly so that resultant force = zero ; 45 3 m s–1
Answers Do not award mark for just ‘area’ without
[2]
(e) –7.95 J 46 7.6 m s –1 reference.
(f) 0.35 J (should be 0 J) calculation of 2 × 105 × 8 = 1.6 × 106 J; [2]
47 10 m s–1
354 (a) P (b) thermal
2 (a) statement energy from
(implication) thathot
workreservoir
done is
m 48 (a) 9.4 × 10 –4
associated m s
3 –1
with area within the rectangle;
D 1.0 C = 1.8 × 10 + 1.6 × 10 J 6 6
200 kPa m
0.8 70° (b) 0.33 Domnot s–1award mark6for just ‘area’ without reference.
= 3.4 × 10 J; 5
R (c) 504.4calculation
kPa of 2 × 10 × 8 = 1.6 × 106 J; (2)
effi ciency = hot reservoir work done
(b) thermal energy from thermal energy from hot reservoir
100 kPa S
A
10° B 49 (a) 3.53 × 10
= 1.8 ×=10 m+×s1.6
–4 6 3 –1 6
1.6 10 × 106 J = 3.4 × 106 J;
F 3.4 × 106 work done
________________________
0 . 5m (b) 4.5​em s–1
fficiency = ​
  
    ​​
base of spine = 47%; thermal energy from hot reservoir [2]
250 cm3 500 cm3 V (c) 290​=kPa_ 1.6 × ​10​​  6​ 1.6 × 106
 ​ [0] 6for ​ = 47%​; (2)
3.4 × ​10​​  ​ = 89%.
(b) 600
(b) 600 K,
K, 1200
1200K, K,600
600KK F 1.6 1.86​× 106
× ​10​​ 
50 21.5 m s[0] for ​=  ​
–1 _
6 ​ = 89%​
(c) 25 J R
(c) closed cycle
1.8 × ​10​​  ​ of rough approximate shape;
(c) 25 J S (c) closed cycle of rough approximate
(d) 50 J 51 18 kPa quality of diagram (adiabaticshape; quality
‘steeper’ than
(d) 50 J
(e) –25 J(c) use of torque = force × perpendicular of diagram
isothermal (adiabatic ‘steeper’
etc.); than isothermal [2]
52 6.5 × 10etc.)(2)
–5
m3 s–1
5 (a) –25
(e) 245 JJ distance;

pressure
(b) 50 J J 53S); 1.97 m s–1
to give torque = S sin (70°) × 0.8 (= 0.752
36 (a) 245
(c) 320 J isothermal
(b) 50 J 54[2] 950 kg m–3
(d) 135 J adiabatic
320 (d) correct balance of torques;
(c)
(e) 185 JJ 55 6.3 × 10 m s–5 3 –1

(d) 135 J F sin (10°) × 0.5 = S sin (70°) × 0.8


6 (a) 525 K
56 9.8 adiabatic
(b)
(e) 360
185 JK to give F = 0.8 sin (70°) = 8.66 ≈ 9; [2]
S 0.5 sin (10°)
(c) 0.31 57 (a) D is same length as A so resonance – this
[Total 8 marks]
37 (a)
(d) 525
1.4 JK implies a π/2 phaseisothermal
difference.
(b)
(e) 2
360 (a)
4.8 JK The radius should be marked as 2 m.
B is much shorter so driver has lower
(f)
(c) 0.29
0.31 Moment of inertia = Idisc + Ichild = 12 md r2 + mcr2
frequency – it will be in phase.
volume
(i) J–1.25= J0.5 K–1 × 60 × 2 + 40 × 2 ;
2 2
7 (d)(a) 1.4 F is much longer so driver has higher
(ii) 2 J KI–1= 280 kg m2; [2] (d) (i)(d) adiabatic (expansion
(i) adiabatic and contraction);
(expansion and contraction);
(e) 4.8 J –1 frequency – it will have
isothermal a π phase
(expansion difference.
and contraction); (2)
(b) 0.75 (b) J K L = Iω = 280 × π = 880 kg m2 s–1 [1] isothermal (expansion and contraction);
(f) 0.29 (ii) correct ‘sense’ of adiabatic
C and E will be somewhere in between. followed by
8 otherwise(c)entropy [2]
I1ω1 =would
I2ω2 be reduced isothermal etc.; e.g. adiabatic (expansion)
(b) D has highest amplitude as it resonates with
then isothermal (contraction) then adiabatic
Practice questions I2 = 0.5 × 60 × 22 + 40 × 12 = 160 kg m2 the driver.(contraction) then isothermal (expansion)
I1ω1 880
2 1 (a) as depth ωincreases, 2
= I =pressure
160
= increases;
5.50 rad s –1
[3] then correct identification of adiabatic as
volume and pressure
2
are inversely proportional the steeper curve when compared with
1 2
(d) KE
for an ideal gas;2 = Iω ; (2) isothermal(2)
M10_IBPH_SB_IBGLB_9021_ANS.indd 2 22/07/2014 16:35
(b) The pressure InitialatKE = 0.5
depth × 280
is equal π2sum
to×the = 1380
of J; 3 (a) adiabatic transformations involve no loss or gain
atmospheric Finalpressure
KE = 0.5 and × the
160pressure
× 5.52 =difference:
2420 J; of heat; the expansion is too fast for heat transfer (2)
​p = A + ρgh​ (b) 0.008 moles (2)
nRT Change nRT in KE: 2420 – 1380 = 1040 J; [3]
_ _
​V = ​ p ​= ​ρgh + A ​​ (c) 5 K 3
(2)
VA _ (d) 0.52 J (2)
​h = _​nRT ​ − ​_
Vρg Vρg ρgV ρg
​= ​nRT ​ − ​_
A ​​ (3)
(e) (i)  (1)
104
(c) some attempt made
M10_IBPH_SB_IBGLB_9021_ANS.indd 3 to compare the equation to ​ 300 K 22/07/2014 16:35

y = mx + c​; 295 K
​​V1 ​​
​h​is linearly related to __ (2) 100

(d) ​± 0.0008 ≈ ±0.001​cm−3(2) 200 205


(e) Temperature (1) (ii) The work done is the area under the graph.
h​values are measured too small; which means
(f ) ​ Any valid method to calculate the area including the
the magnitude of the intercept is too large (2) full extent of the pressure
0.5 J (3)

14
Answers

(f ) The temperature change is too fast for accurate (e) Energy is removed from the gas and so entropy
measurements.(2) decreases OR temperature decreases at constant
4 (a) (i) on – gas is compressed (1) volume (less disorder) so entropy decreases. (1)
Correct answer and correct explanation. (f ) Different paradigms/ways of thinking/modeling/
views OR allows testing in different ways OR
(ii) ejected from – pressure remains constant,
laws can be applied different situations. (1)
volume reduced so temperature must go
down(1)
Correct answer and correct explanation.
(b) work done = pΔV = –1.0 × 105 × 0.4 B.5
= –0.40 × 105 J (40 kJ); (2)
Sign should be consistent with (a) (i) above. Work ‘by’ and Exercises
+ work would get zero for (a) (i) but [2] marks here.
1 3.7 × 10–4 m
(c) area enclosed; 0.6 (±0.2) × 105 J (60 kJ ± 20 kJ); (2)
2 10.8 Ω
​work
(d) ​efficiency = _ out
heat in
​​ ;
60 3 3 kΩ
​= ​_
120
​= 50%​(± 17%)​​; (2)
4 0.3 V
5 D(1)
5 0.02 A
6 A(1)
6 100 Ω, 100 Ω, 25 Ω
7 (a) 0.297 OR 29.7% (2)
7 1Ω
Award [1 max.] if temperatures are not converted to K,
giving result 0.430. 8 11.5 V
(b) the Carnot efficiency is the maximum possible 9 (a) 500 J
OR the Carnot cycle is theoretical/reversible/ (b) 3 × 104 J
impossible/infinitely slow; energy losses to
10 0.031 W
surroundings due to friction/electrical losses/
heat losses/sound energy (2) 11 0.5 W
(c) 1.79 × 1013 J. (3) 12 (a) 450 kJ
Allow solution utilizing wasted power 78.9% (b) 37.5 kW
Award [2 max.] if 71% used as the overall efficiency (c) 125 A
giving an answer of 1.96 × 1012 J.
13 no energy is lost, no heat produced, motor is 100%
Watch for ECF from (a). efficient, no friction / no other losses
(d) Law 1: net thermal energy flow is QIN – QOUT in
14 (a) 0.45 A
which QOUT refers to “waste heat”;
(b) 20 J
Law 1: QIN – QOUT = ∆Q = ∆W as ∆U is zero;
Law 2: does not forbid QOUT = 0; 15 (a) 4.5 A
Law 2: no power plant can convert 100% of QIN (b) 1.8 × 107 J
into work; 16 (a) ​​__
16
3 ​​ Ω
Law 2: total entropy must increase so some Q
(b) 8 Ω
must enter surroundings (3)
(c) 28 Ω
Max 2 marks if only one law is referred to. 16
(d) ​​__
7 ​​ Ω
8 (a) 52.6 m3 OR 52.7 m3(2)
(b) 62.7 × 103 J(2) 17 0.44 V
(c) 1.57 × 105 J(3) 18 0.49 V
(d)  (2)
p

0
0 V

15
Answers

Practice questions (d) energy from Sun/photovoltaic cells is renewable


OR non-renewable are running out;
1 (a) v  (1) non-polluting/clean; no greenhouse gases OR
does not contribute to global warming/climate
change (max. 2)
​​2.0 7× ​10​​  ​​= 2857 ≈ 3000 s​
4
9 (a) time taken = _ (1)
E
Must see at least 2 s.f.
(b) (i) when electrons collide with the lattice atoms;
(b) Electrical energy = ​qV​= 43 × 103 × 16
they lose energy (2) = 6.88 × 105 J
(ii) when electrons collide with the lattice
​​Et ​= _
​6.88 ×  ​10​​  ​ 
5

atoms, the lattice atoms gain kinetic energy; Power = _ 2857


​= 241 ≈ 240 W​ (2)
increased kinetic energy means increased Must see at least 3 s.f.
temperature(2) (c) use of power = force × speed OR
(iii) the more the lattice atoms vibrate due to force × distance = power × time; 34 N (2)
increased kinetic energy, the more the (d) 34 N (1)
electrons will collide with them; (e) −1
3.5 m s (2)
the resistance increases after being Award [0] for solutions involving use of kinetic energy.
switched on (2) Award [0] for v = 7 m s –1.
15q Award [2] for a bald correct answer.
I=_
(c) ​ ​ t ​​
the time taken by the right-most charge carrier to (f ) the maximum distance will decrease;
pass through the left is ​​_L ​​ because opposing/resistive force has increased
v
15q _ 15vq OR because more energy is transferred to
_
substituting ​I = ​ _L ​= ​ L ​​ (3)
​  ​
gravitational potential energy OR because
v
2 (a) (i) 140 ​Ω​ (2) velocity has decreased OR increased mass means
(ii) 0.06 A (2) more work required to move up the hill (2)
(iii) 0.5 W (2) (g) 0.62 Ω(2)
(iv) 726 K (2) V dropped across battery OR Rcircuit = 1.85 Ω;
(b) The wire is not a perfect black body. (1) For MP1 allow use of internal resistance equations that
(c) ​P = ​I​​  2​  R = 0. ​06​​  2​ × 1 = 0.0036 W​ lead to 16 − 12 = 4 V.
​E = Pt = 0.0036 × 60 = 0.216​J (h) 3.2 V (1)
Q (i) 0.25 Ω(2)
​Q = CΔT​so ​ΔT = _ ​0.216
​C ​=_ 10
​= 0.0022 K​
10 (a) 0.32 m (1)
This increase in temperature is insufficient to
(b) 0.068 A (2)
damage the cell. (3)
Award [1] for 4.3 A where candidate has not calculated area
3 A(1) (c) Quantities such as resistivity depend on the
4 D(1) material OR they allow the selection of the
5 A(1) correct material OR they allow scientists to
compare properties of materials. (1)
6 B(1)
(d) as area is larger and length is smaller;
7 C(1) resistance is very much smaller (2)
8 (a) emf is defined as the terminal potential difference Award [1 max.] for answers that involve a calculation
when no current flows; in this instance, a current (e) complete functional circuit with ammeter in
is present and so the potential difference will be series with resistor and voltmeter across it;
less than the emf due to ‘lost volts’ in the internal potential divider arrangement correct
resistance;(2)
 (2)
(b) ​7.2 Ω​ (3)
multiple solutions are possible A
Award [3] marks for a bald correct answer
(c) 0.12 OR 12% (3) V

16
Answers
Answers
Chapter 5
(c) (c)3π Use of ​a = − ​ω​​  ​  x​
2

C.1 Exercises 2
​ω = ​_ 2π _
​​π4 ​  rad ​s​​  −1​​
​= ​ 2π​= 0.795​or _
T 7.9
20 170 Hz
​​x0​  ​​= 4.1 m​; (allow answers in the range of 4.0 to 4.25 m)
Exercises
1 b 21 (a) 33.2 two cmsignificant figures
(b) 2in final answer whatever
1 Only (b) is an example of simple harmonic motion. the value (4)
2 (a) 1.67 Hz (b) 10.5 rad s–1 22 a lower
(a) (d) note on the way down,
shape correct, constant amplitude for new and a higher
2 (a) 1.67 Hz (b) 10.5 rad s –1
note curve,on the way upof 10 s shown; (there must be
minimum
3
3 1.1 cm down
1.1 cm down
(b) 1100 some Hzconsistent lead or lag and no change in T)
4 displacement lead/lag of 1 s (to within half a square by eye);
(c) 892 Hz
3 cm 3  (2)

a/m s−2
(d) a higher 2
note on the way down, and a lower
original
+π/4
note on the way up
time 1 −π/4
5s
23 331.5 m s 0 –1
t/s
0 2 4 6 8 10 12
24 317.6 Hz −1
5 1.67 ss −2
1.67 25 29°
6 0.5 m s–1 downwards −3
6 0.5 m s–1 downwards 26 815°(a) acceleration/restoring force is proportional to
7 (a) 2.5 m s–1 (b) 3.16 m s–2
7 (a) 2.5 m s–1 (b) 3.16 m s–2 displacement;
8 0.62 m s–1 m s–1in the opposite direction/directed toward
27 2 × 108and
8
9 0.62 m s–1
0.32 m 28 42° equilibrium;(2)
10 0.32
(a) 3π –1
(b) 7.1 × 10–3 J (b) 0.38 kg / 0.39 kg(2)
9 m rad s 29 27° ​T​  2​  ​ ​m1​  ​​ _
(c) 7.1 × 10–3 J
(e) 3π
10 (a) 1.8 rad
× 10s–3–1 J
(d) 5.3 × 10–3 J
(b) 7.1 × 10–3 J
Use of either _
38.8°
30 (a) (c)
​​ 12 ​= _
​T2​  ​  ​
​​m​  ​​ ​​ OR ​T = 2π ​
(b)
2
51.2°
 ​ 
√m
_
k
​  ​​
0.032 J (3)
(c) 7.1 × 10–3 J (d) 5.3 × 10–3 J (c) 41.8°
Allow ECF from (b) (d) yes
Challenge yourself and incorrect ω.
(e) 1.8 × 10 J
–3
Allow
(e) 62.2 μm answer using k from part (b).
1 T = 0.6 s (d) spring constant/k/stiffness would increase;
11 280 m s–1 31 0.11 m T would be smaller;
Practice questions
12 (a) 0.2 m (b) inverted 32 88 μm fractional uncertainty in T would be greater, so
1 D(1) fractional uncertainty of mass of block would be
(c) Some of wave is reflected so energy in 7greater;(3)
2 A(1)33 7 × 10 m
transmitted wave is less.
3 C(1)34 914.6 (a) the restoring force/acceleration is proportional to
m
13 1.2 m s –1 displacement(1)
4 D(1)
Allowrad
35 No, 0.0001 use <of 0.00014 ​F ∝ − x​or ​a ∝ − x​
symbols i.e.rad (1)
14
5 (a)
B(1)182.6 m s–1 (b) 143.8 Hz
(b) ​4.43 rad ​s​​  −1​​
6 B(1) 36 9 cm
15 54 N evidence of equating ​m ​ω​​  2​  x = ρAgx​ to obtain
7 (a) force/acceleration proportional to the 37 6 cm ​​ ρAg
_
m ​= ​ω​​  ​​
2
(2)
16 812.5 Hz
displacement/distance from a (fixed/equilibrium)
Answer to at least ​3​s.f.
point/mean position; 38 (a) 3.3 μm (b) 12.2°
17 (a) 1.7 Hz (b) 0.24 m (c) 71.4 J OR 72.4 J (2)
directed toward this (equilibrium) point / in
(c) 24° 39 982 lines
(depending on use of 4.4 or 4.43)
opposite direction to displacement/ distance;
18 35° Allow algebra only if symbols are fully explained.(2)40 150 nm
(b) 0.73 N (1)
19 π
(a) allow (b) π 41 95 nm
answer in range of 0.71 to 0.75 N
5

PH_SB_IBGLB_9021_ANS.indd 1 16/07/2014 09:37

17
Answers

(d) • ultrasound for medical imaging and the reasons


kinetic energy

that we seek knowledge (e.g. about our body).


Ek max
• gamma waves for sterilisation or radiotherapy,
how they are detected alternatively as photons
under certain conditions, and constraints on
0 knowledge.
0 time
T
Practice questions
1 (a) longitudinal (1)
(b) (i) wavelength = 0.5 m (1)
energy never negative; (ii) amplitude = 0.5 mm (1)
correct shape with two maxima; (2) (iii) correct substitution into
10 (a) 1.054 Hz; 0.953 Hz (2) speed = frequency × wavelength;
(b) 0.10 Hz; 10 s (2) to give v = 660 × 0.5 = 330 m s−1;(2)
2 (a)  (1)
land

C.2 P
Q

Exercises deep water channel

1 280 m s–1 (b) The waves from the shallow water and the
2 (a) 0.2 m (b) inverted channel superpose. They interfere constructively.(1)
(c) Some of wave is reflected so energy in (c) The wavelength decreases. This is because the
transmitted wave is less. speed of the wave decreases but the frequency is
constant.(1)
3 1.2 m s–1
(d)
Challenge yourself
1 Your report on X-rays is likely to include ideas about Energy is conserved, and so all must remain
William Conrad Röntgen in 1895, cathode rays (beams contained within the same, shorter, crest. (1)
of electrons), a fluorescent detector and the idea of 3 v = 0.5 × 14 = 7 ​cm s (3)
(a) (i) ​ −1
the discovery being by accident. You should also 2π
(ii) ​​_8
​​ OR 0.79 rad. (2)
have included the references that you made use of in
formulating a response. Connections to TOK include (b) (i) 14 cm (1)
Röntgen’s scope of work being related to the scientific (ii) 0.71 Hz (1)
concepts of matter, waves and energy, atypical method (iii) 9.9 cm s−1(1)
(i.e. not following a scientific method), typical use of 4 (a) (i) distance traveled per unit time; by the energy
tools (i.e. apparatus for measuring), perspective in the of the wave / by a wavefront; (2)
sense of convincing others of his work’s significance
(ii) velocity has direction; but light travels in all
and verifiability, and ethics in unknowingly making
directions;(2)
the first published step into understanding ionizing
radiation and its uses and risks. (b) (i) distance in a particular direction; (accept in
terms of energy transfer) (of a particle) from
Your report on waves being used in technology to
its mean position; (2)
improve society is inherently connected to TOK
because technology is one of the themes on which (ii) longitudinal: displacement along;
your exhibition can be based – provided that you have transverse: displacement normal to;
connected your response in some way to knowledge. direction of transfer of wave energy /
Examples might include: propagation, not motion; (3)
• radio waves for communications and how this Award [0] for left / right and up / down for
relates to the ownership of knowledge through longitudinal / transverse.
the editing of radio station content.

18
Answers

(c) (i) ​​(_ 125 )


​  1200
 ​ ​ = 9.6 km ​s​​  −1​ ​(± 0.1)​​ (1) (ii) molecules to the left of the particle have
moved left and those to the right have
(ii) ​​(_ 206 )
​  1200
 ​ ​ = 5.8 km ​s​​  −1​ ​(± 0.1)​​(1)
moved right;
Award [1] if the answers to (i) and (ii) are given in hence the particle is at the center of a
reversed order. rarefaction;(2)
(d) (i) P shown as the earlier (in-between) pulse (1)
D = 6052​km
17 (a) (i) ​ (2)
(ii) laboratory L3(1) D ​= _
T=_
(ii) ​ ​speed ​6052
800
​​ hours
(iii) e.g. pulses arrive sooner; smaller S–P
interval; larger amplitude of pulses; (3) This is 7 hours 34 minutes. (2)
Allow any feasible piece of evidence, award [1] for (b) After arrival of the S wave, the time delay
each up to [3]. between the P and S waves can be determined
(iv) distance from L1 = 1060 km (± 20) and D can be calculated. The time before the
distance from L2 = 650 km (± 20) arrival of the tsunami (i.e. the warning time) is:
distance from L3 = 420 km (± 20) ​  ​= (
t​​ W ​_
​  800 3.00 × 60 × 60 )
D  ​  − ​ ___________
D  ​ ​​ hours for D measured
Accept 3 significant figures in all three estimates. in km OR ​​tW ​  ​= 1.16 × ​10​​  −3​  D​ hours. (2)
some explanation of working; (4)
(v) position marked, consistent with answers to
(iv); to the right of line L2L3, closer to L3(1) C.3
If the answers given in (iv) mean that the point
cannot be plotted, then only allow the mark if the
candidate states that the position cannot be plotted /
Exercises
does not make sense. 1 (a) 1.7 Hz (b) 0.24 m
5 B(1) (c) 24°
6 A(1) 2 35°
π
7 C(1) 3 (a) ​​ __5 ​​ (b) π

8 C(1) (c) ​​__
2 ​​
9 C(1) 4 29°
10 C(1) 5 15°
11 D (1) 6 2 × 108 m s–1
12 B(1) 7 42°
13 D(1) 8 27°
14 C(1) 9 (a) 38.8° (b) 51.2°
15 A(1) (c) 41.8° (d) yes
16 (a) a wave where the displacement of particles/ (e) 62.2 μm
oscillations of particles/movement of particles/ 10 0.11 m
vibrations of particles is parallel to the direction of 11 88 μm
energy transfer/wave travel/wave movement.
12 9 cm
Do not allow “direction of wave”.(1)
(b) (i) 340 m s−1(2) 13 6 cm
Either v = f λ OR distance traveled ÷ time
Practice questions
​T1 ​​)
(ii) 213 Hz ( v = f λ) OR 210 Hz (​F = _ (2)
(c) (i) the displacement of the particle decreases 1 (a) (i) the phase difference between light leaving S1
OR on the graph displacement is going in and S2 is constant (1)
a negative direction OR on the graph the Do not penalize candidates if they state ‘has the
particle goes down; same phase’.
to the left; (2) (ii) to produce sufficient diffraction; for the
Do not allow “moving downwards” unless beams to overlap; OWTTE(2)
accompanied by reference to graph.
19
working; [4] Do not penalize candidates if they state (b) λ
‘has the same phase’.
Answers
nsistent with
(ii) to produce sufficient diffraction;
A
L3, closer to L3; [1] for the beams to overlap;
in (iv) mean that OWTTE; [2] M
plotted, then only path difference between S1 and S2 is an
(d) (i)
candidate states
(b) (i) integral
path difference numberSof and
between wavelengths;
S is an [1] (b)
(b)
1 2 λ
nnot be plotted / Accept
integral number of‘waves arrive at P in phase’.
wavelengths (1)
e.
(ii) arrive at P in phase’.
Accept ‘waves
node at centre, A
d;
(ii)
[1]
intensity N
M
ing wave = (i) downwards; [1]
(ECF )
(ii) correct marking of A; [1]
(iii) correct marking of λ; [1] (
03) ≈ 6 Hz (iv) positive sine curve;
O P distance along the screen
correct position of N; [2]
cy is natural N
maximum atmaximum
O and P; Watch for ECF from (i).
n of at O and P; shape with
general (i) downwards (1)
f resonance / minimum about
generalhalf way between
minimumOabout and half
P (2) (i) downwards;
(i) f = v = to give 2.0 Hz; [1][1]
le if ECF ); (c) [3]
shape with (ii)(c)correct marking
λ of A (1)
fringe spacing = way 10–4 m;
2.5 ×between O and P; [2] (ii) correct marking of A; [1]
[Total 25 marks] ___________________
(iii) correct = 0.5 s; of λ(1)
(ii) T marking
​(2.5 × ​10​​ 
(e)−4​ × × ​10​​  −3​)​= 2.5 × 10–4 m;
3.00spacing
fringe
​λ = ​  
1.50 ​ = 5.0 × ​1–3 0​​  ​  m​ −7
(2) correct
(iii) positive
(iv) vT curve;
s =marking
sine λ; cm;position of N;
of correct
= 1.25 (1.3) [1] (2)
λ = (2.5 × 10 × 3.00 × 10 ) = 5.0 × 10–7 m;
–4
f the medium/ 4
2
waves travel); (a) ray: direction in which wave 1.50 (energy) is traveling; Watch
(iv) positive
or for ECF from
sine curve;(i).
[2]
hree
dividual (c)wavefront:
(i) the phase line joining
difference (neighboring)
between light points that (c) (i) ​ = _​λv ​​toingive
fcorrect T 2.0 Hz
position of N; forward 1 λ;
wave moves [2] (1)
hree (c) (i) the phase difference between light [Total 14 marks]
have the leaving
same S1 and phase S2 is/ constant;
displacement; [1] or suitable 4 _ 4 (
​vT
d by each wave); [2]
leaving
Wave
S1 and S2 is constant;
properties
[1] (ii) TWatch
= 0.5 s; for
5​s =ECF ​​=from
1.25 (i).
(1.3) cm
[4]
derstanding and [1] reference 5 not
Do to Huygens’
penalize principle;
candidates if theyray is
state normal to a 4
= = 1.25 (1.3) cm; [2] 6
[4] Do not penalize candidates if they state 4
‘has (a)
wavefront(3) (i)
the same
‘has the same phase’.
phase’. [1] (c) (i) (d)OR
f =Principle v = to give
of 2.0 Hz;
superposition: [1]
uld be identical. (ii) to
(b) (i)(ii) wavefront produce sufficient diffraction;
parallel diffraction;
to D (1) in _​​  T4 ​​λwave moves forward _ ​​  14 ​  λ​;
to produce sufficient (ii) T _5= when two or more waves overlap; the
0.5 s;
L3; [1] for the beams to overlap;
L3; [1] (ii) frequency
for the beams to overlap; since v = f λ,
is constant; ​= ​4resultant
​= 1.25 (1.3) cm​;
displacement at any point (2)
hat OWTTE; [2] s =is the vT = 1.25 (1.3)displacements
cm;
hat vOWTTE;
∝ λ; wavelength longer in medium [2] I, sum of the
(d) Principle of4superposition: when two or more waves due to
only (d) (i) path difference between S1 and S2 is an
only (d) (i) hence higher speed
path difference between inSmedium
1
and S2
isI;
an (3) or
overlap; each the wave
resultant separately / OWTTE;
displacement at any point
ates B integral number of wavelengths; [1]
ates integral
Allow numberbased
solution of wavelengths;
on angles marked [1] on diagram T wave 1
d/ Accept ‘waves arrive at P in phase’. is theinsumAward of the displacements
for
[2]moves forward λ;to each
an answer that due shows wave
a clear
d/
orAccept
speed of ‘waves
(ii)wavefronts. arrive at P in phase’. [1] separately 4 / OWTTE; of the principle,
understanding 4 [1] for a
(ii)
e, (ii) V

_ ​  ​​ _λ
​ ​  ​​ Award= [2] 5 = for1.25
an answer
(1.3) that
cm;shows aand clear[0] for a [2] 6 (
e,
I I
intensity (iii) ratio = ​​​V​  ​​ ​= ​​λ​  ​​ ​​(or based on Snell’s law);
reasonable understanding
[1] intensity R R understanding 4 of the principle, [1] for a reasonable
[1] (d) Principle weak of answer.
superposition:
X; 3.0
_
​= ​ 2.5 ​ = 2.0​ allow ± 0.5(2) understanding and [0] for a weak answer.
Y; Explanation:
when two or more waves overlap; the
each wave; (c) (i) velocity / displacement / direction in (+) and
Explanation:
resultant displacement at any point
is two divisions; [4] (–) directions; idea of periodicity (2)
1 is the sum of the displacements due to
(ii) period = 3.0 ms; frequency = ​​_
T
​​= 330 Hz (2) =
each wave separately / OWTTE;
(iii) Accept any oneP ofdistance
the following. 3
O along the screen
Award [2] for an answer that shows a clear (
O distance along the screen
at time t = 0, 1.5P ms, 3.0 ms, 4.5 ms, etc.(1) suitable diagram;
suitable diagram;
maximum at O and P;
maximum at O and =P;140 ± 10 squares / mean understanding
when two
when twoof
positive the
pulses
positive principle,
(or two
pulses (or[1]
wave
twoforwave
a
crests)
(iv) area of half-loop
general shape with minimum about half
16/07/2014 09:38

[3] vgeneral
= 4.0 mshape
s–1 with minimum about half
accept ± 0.2; overlap,
reasonable they reinforce
crests)understanding / OWTTE
overlap, they reinforce and [0]/ OWTTE; for a [4] (4)
[3] way between O and P; [2]
marks]
way between O and P; –3
= 140 × 0.4 × 0.1 × –4–410
[2] Any
weak situation
answer. where resultant displacement looks as
marks] (e) fringe spacing = 2.5 × 10 m;
(e) fringe spacing = 2.5 × 10 m;
–4 –3 m
though it is the sum of the individual displacements.
/ = 5.6××1010
λ = (2.5 × 3.00 × 10–3 ) 5.0 × 10–7 m; (2) Explanation:
/
–4
× 3.00 × 10–3) =
λ = (2.5 × 10 1.50 = 5.0 × 10–7 m; Mark the description of the principle and the description
; Award [1]1.50for area of triangle. [2] of constructive interference together.
; [2]
(v) (twice) the amplitude; [Total 14 marks] (1) (e)4 (i) S2X = nλ;
ve); [2] [Total 14 marks]
ve); [2]
5
Allow
Wave properties
distance moved in 1.5 ms. where n = 0, 1, 2; (Accept = ‘n is an integer’)(2) (
and [1] 5 Wave properties
and [1] 3 (a) (a)(i)(i) (1) ​S​  ​​  X
[1]
(a) (i) [1] (ii) ​sin θ ≈ θ​; therefore ​θ = _
M05_IBPH_SB_IBGLB_9021_ANS.indd 4
​ 2d ​​ (2)
al.
al. suitable ​ydiagram;
​  ​​
(iii) ​ϕ = ​_
D
n
​​ (1)
when two positive pulses (or two wave
Award the small angle approximation mark
crests) overlap, they reinforce / OWTTE; [4]
anywhere in (i) or (ii).
B
B

(ii) [1]
(ii)(ii) (1)
[1]

4 20
Answers

​S​  ​​  X

(f ) (i) ​θ = ​_
2
d
​= ​_d
​dθ
​  so λ = _n ​​; I/I0
6
substitute to get λ = 4.73 × 10–7 m (2) 5

θ​and ​ϕ​are small;


(ii) ​ 4
​y​  ​​
​​ dλ ​= _
therefore _ Dλ
​Dn ​  so y = ​_
3
d
​ = 0.51 mm​ (3)
2

4 C(1) 1

5 D(1) −4 −2 0 2 4 x/cm

6 C(1) (c) As the slit separation increases, the fringe


7 A(1) spacing decreases. (1)

8 D(1) 16 (a) there is constructive interference at M OR the


amplitude doubles at M;
9 D(1) intensity is proportional to amplitude;
10 C(1) 88 W m−2(3)
11 B(1) (b) 560 nm (2)
(c) The interference pattern will be modulated by
12 A(1)
single-slit diffraction envelope and so it will be
13 B(1) less.(1)
sin i _____________ (d) ALTERNATIVE 1
​3 × ​10​​  ​ × sin​33° ​
8 ( )
v = c ​_
14 (a) ​ sin r
​=    ​​= 2.3 × 108 m s–1(2)
( sin​ 46° ​
)
the angular position of this point is
(b) ALTERNATIVE 1
​28 1.5
​θ = _ × ​10​​  −3​
​ = 0.01867​rad, which coincides with
light strikes AB at an angle of 57°;
critical angle = 50.1°; the first minimum of the diffraction envelope;
​bλ ​= _
​ 560 × ​10​​  −3​​ = 0.01867​rad, so intensity will
−9
49.2° from unrounded value ​θ = _
0.030 × ​10​​  ​
angle of incidence is greater than critical angle so be zero (2)
total internal reflection (3)
ALTERNATIVE 2 ALTERNATIVE 2
the first minimum of the diffraction envelope is at ​
light strikes AB at an angle of 57°;
θ = ​_bλ ​= _
​ 560 × ​10​​  −3​​ = 0.01867​rad;
−9

calculation showing sin of “refracted angle” = 1.1; 0.030 × ​10​​  ​


statement that since 1.1 > 1 the angle does not distance on screen is ​y = 1.50 × 0.01867 = 28​mm
exist and the light does not emerge (3) so intensity will be zero (2)
(c) total internal reflection shown; 17 (a) 0 OR 2π radians OR 360° (1)
ray emerges at opposite face to incidence (2)
λ​
(b) 4​ (1)
Judge angle of incidence=angle of reflection by eye or XZ 4λ
accept correctly labeled angles (c) ​sin θ = _
​VX ​= _
​2d ​ ⇒ 2λ = d sin θ​ (1)
With sensible refraction in correct direction (d) identifies gradient with _ ​​dλ ​​ OR use of ​d sin θ = nλ​;
λ
15 (a) There is no diffraction at either lamp because ​d = ​_
0.08
​6330.08
​= _ × ​10​​  −9​
​ = 7.91 × ​10​​  −6​  m​
there are no slits. Therefore, the intensity has no
126 mm−1 or 127 mm−1(4)
pattern.(1)
(e) (i) Gradient would decrease. (1)
I/I0
6 (ii) No change (1)
4 18 Path c (1)

2
The lifeguard can run faster on the beach than
swimming in water. Of the three options, path c has
the shortest distance in the water and so will take the
0 x
least time. (1)
(b) This is the standard two-slit pattern.
The maximum intensity is four times the 19 A(1)
separate intensities because the amplitudes
sum vectorially. The fringe spacing is calculated
using _​​ λD
d
​​ to be 2.0 cm. (3)

21
4 (a) the net displacement of the medium/ λ = (2.5 × 10

Answers particles (through which waves travel);


is equal to the sum of individual
displacements (produced by each wave); [2]
Award [2] for a good understanding and [1] 5 Wave properties
for a reasonable one. (a) (i)
20 The resultant wave form is the sum of the two (b) Wave X
(b) Xand
andwave
waveYYshould
shouldbebeidentical.
identical.
wave forms. X and Y
(4)

C B OA C B AO
A B

(ii)

C B A C B A correct phase
correct phase forfor wave
wave X; X; correct phase for wave
O O
correct
Y; phase for the
amplitudes wavesame
Y; for each wave; amplitude
amplitudes
for the same
each wave for each
is two wave;
divisions (4)
amplitude for each wave is two divisions; [4]
2 (a) illustration showing node at center, antinode at
C
O
B A C
O
B A each end (1)
(b) wavelength of standing wave = (2 × 280)
= 560 m / (ECF )
M05_IBPH_SB_IBGLB_9021_ANS.indd 3
OR
C.4 _​​ 3.4 6× ​10​​  ​​ = 570 m​;
3

​3.4560
× ​10​​  ​
3
​frequency = _ ​≈ 6 Hz​
Exercises
earthquake frequency is natural frequency of
1 (a) 182.6 m s–1 (b) 143.8 Hz vibration of building / mention of resonance /
2 54 N multiple / (submultiple if ECF) (3)
3 812.5 Hz 7
3 A A 8 (a) (i)
(ii)
4 170 Hz
5 (a) 33.2 cm (b) 2
6 (a) D is same length as A so resonance – this implies (b) (i)
a ​​ _π2 ​​ phase difference.
B is much shorter so driver has lower frequency – N
(ii)
it will be in phase.
F is much longer so driver has higher frequency –
it will have a π phase difference.
C and E will be somewhere in between.
(b) D has highest amplitude as it resonates with the
driver.
A N

Challenge yourself pipe A pipe B


(a) (i)(i) correct
(a) correctwave
waveshape
shapefor
forpipe
pipeA;
A; correct wave
2πx λ 3λ
1 y = 2A cos ​_ _ _
λ ​  sin (ωt)​ zero when x = ​​4 ​​, ​​ 4 ​​ etc.
shape for
correct pipe
wave B for pipe B;
shape [2] (2)
​ (ii) correct
correctmarking
markingofofA A andN Nforfor pipe
C
(ii) and pipe A;A;
Practice questions
correct marking of A and N for pipe B;B
correct marking of A and N for pipe (2)
(b) (i) for pipe A, λ = 2L, where L is length of 1 T = 0.6
1 (a) the net displacement of the medium/ particles [2] the pipe;
_c
(through which waves travel); is equal to the sum c = f λ to give ​L = ​2f ​​ ; 2 y = 2A
(b) (i) for pipe A, λ = 2L, where L is length of
of individual displacements (produced by each substitute
the pipe; to get L = 0.317 m; (3)
wave)(2)
(ii) cfor
= f32 Hz,
λ to = c ;pipe will have a length
theLopen
give
Award [2] for a good understanding and [1] for a of about 5tom;
2f
substitute getwhereas
L = 0.317 them;closed pipe [3]will
reasonable one. have half this length, so will not take up as
(ii) for 32 Hz, the open pipe will have a
much space as the open pipe / OWTTE(2)
length of about 5 m;
whereas the closed pipe will have half
this length, so will not take up as
much space as the open pipe /
OWTTE; [2]
22
Answers

1 1
The argument does not have to be quantitative. (b) Using ​ω = ​_ _ ​= ____________________
​  
_______________________ ​
​√LC ​ ​√4.00
   × ​10​​  −3​ × 2.00 × ​10​​  −12​ ​
Award [1] for recognition that low frequencies mean
longer pipes and [1] that for the same frequency, = 1.1 × ​10​​  7​​ Hz
closed pipes will be half the length of open pipes. The
​ω ​= _
​1.12π
× ​10​​  ​
7
fact they need less space can be implicit. ​f = _

​= 1.8 × ​10​​  6​​ Hz (2)
4 (a) (i) the motion corresponds approximately to the (c) increases to a maximum; then falls
horizontal displacement of the joint; I (2)
this is sinusoidal, which is analogous to SHM(2)
(ii) because of the angle of the connecting rod;
this is reduced by the connecting rod
being long (so approximately horizontal f
throughout)(2)
6 A(1)
(iii) 1 cm (1)
7 C(1)
8 (a) traveling wave moves along the length of the
string and reflects at fixed end; superposition/
interference of incident and reflected waves; the
2 cm superposition of the reflections is reinforced only
for certain wavelengths (max. 2)
(iv) 2 Hz (1) λ = 2l = 2 × 0.62 = 1.24 m​;
(b) ​
(b) (i) 2 m (1) ​v = f λ = 195 × 1.24 = 242 ​m s​​ −1​​ (2)
−1
(ii) 4 m s (1) Answer must be to 3 or more s.f. or working shown for
(iii) the wave reflects; MP2.
with no phase change (2) (c) straight line through origin with negative
(iv) At the point when the wave reaches the end, gradient(1)
62
a standing wave has not yet been formed; (d) ​​_3
​ = 21 cm​ (1)
only one cycle has passed.
9 (a) A horizontal line shown in center of pipe. (1)
After reflection, a standing wave will form
(b) air molecule moves to the right and then back to
with both ends as antinodes if the length of
the left; returns to X/original position (2)
spring is equal to a whole number of half
wavelengths. (c) wavelength = 2 × 1.4 = 2.8 m;
(3) c = f λ = 120 × 2.8 = 340 m s−1;
λ = 2l
K = ρc2 = 1.3 × 3402 = 1.5 × 105; kg m–1 s–2(4)
10 (a) energy is not propagated by standing waves;
λ=l amplitude constant for traveling waves OR
amplitude varies with position for standing waves
(c) From Hooke’s law we know that ​​Ft​  ​​= kx​ OR standing waves have nodes/antinodes;
​M
and assuming that ​μ = _x ​​ phase varies with position for traveling waves OR
_ _ _

√​  kx √ √
​  k ​Mx​​  ​​  ​= ​ _ k phase constant inter-node for standing waves;
2
​v = ​ _ M
_
 ​  ​= ​ _ ​  M ​  ​  x​, which means that velocity
​  x ​ traveling waves can have any wavelength OR
and length are proportional. (3) standing waves have discrete wavelengths (2)
(d) Assuming SHM with maximum acceleration and (b) sound wave travels down tube and is reflected;
acceleration equal to g: ​a = ​ω​​  2​ ​x0​  ​​​ incident and reflected wave superpose/combine/
9.81
​​ω​​  2​= _
​0.01 ​ = 981​; interfere(2)
ω
​ = 31.3 = 2πf​; Do not award MP1 if the reflection is quoted at the walls/
​f = 5.0​Hz (3) container
(c) nodes shown at water surface AND _ ​23 ​​ way up
5 (a) if an alternating current is applied to the circuit, tube (by eye) (1)
it will be forced to oscillate; if the frequency is the
same as the first harmonic; then resonance will Accept drawing of displacement diagram for correct
occur; which leads to a large current (4) harmonic without nodes specifically identified.
Award [0] if waveform is shown below the water surface.(1)

23
Answers

λ = 0.74 m​;
(d) ​
​f = _​λc ​= _320
​0.74 ​= 430 Hz​ (2) C.5
11 (a) (i) Amplitude is increasing as energy is added. (1)
(ii) energy input = energy lost due to damping (1) Exercises
(b) curve from time tB reaching zero displacement; 1 (a) a lower note on the way down, and a higher note
in no more than one cycle on the way up
 (2) (b) 1100 Hz
vertical
displacement

tA tB
(c) 892 Hz
(d) a higher note on the way down, and a lower note
0
time on the way up
2 331.5 m s–1
3 317.6 Hz
​25
12 (a) T = _10
​T1 ​​ OR evidence of f = _
​​ = 2.5 s AND f = _ 10
​25 ​​ (1) 4 0.06 c
(b) 30 m s corresponds to f = 1.2 Hz;
–1
5 4.38 nm
the amplitude of vibration is a maximum for this 6 3.17 × 107 m s–1
speed OR corresponds to the resonant frequency(2)
(c) similar shape with lower amplitude; 7 3.25 × 107 m s–1; it is further away since it is moving fast
maximum shifted slightly to left of the original 8 2.1 Mpc
curve(2) 9 1440 km s–1
Amplitude must be lower than the original, but allow the
amplitude to be equal at the extremes Practice questions
13 A(1) 1 (a) circular wavefronts originating from four
14 D(1) successive source positions; bunching of
wavefronts in front, spreading out at back;
15 B The trick is to push the rolling ball at the right times
approximately, correct spacing of wavefronts in
and in the right direction. The mouse would need to
front, and behind source (3)
match the natural rhythm of the back-and-forth of
the ball. (2) (b) f waves in distance (V − v);
apparent wavelength = (V − v);
16 (a) when the pan is moving upwards with a f
retardation when approaching its amplitude, apparent frequency = f × v (3)
the mass will leave the pan if the retarding (V − v)
Allow any other valid and correct approach or statement
acceleration is less than g;
of formula. Award [0] for quote of formula with no
for displacement x, using ​F = kx = ma​, the
kx
working shown.
retarding acceleration is ​− ​_
m ​​ where k is the spring
constant; (c) λʹ = λ(V − v);
V
5999.996 = 600 × (3 × 10
8
kx
​​4 ​π​​  2​  x ​ > g​ − v) ;
2
when ​− ​_ g​, i.e. when _
m ​< − _ ​T​​  ​ 3 × 10
8

(since T
​ = 2π ​
 ​ m
_
k √
​  ​​) the mass leaves the pan;
(b) As the amplitude varies, this will occur first
(3) v = 2000 m s−1(3)
Allow alternative version for red shift.
​T​​  2​  g
​​ 4 ​π​​  2​  A ​ = g​i.e. ​A = _
2
when _ ​ 2 ​​ 2 B(1)
​T​​  ​ 4 ​π​​  ​
Substituting the data provided, 3 D(1)
​0. ​50​​  ​ ×2 9.81 ​= 0.062 m = 6.2​ cm
2
​ =_
A (3) 4 D(1)
4 ​π​​  ​
5 B(1)
6 A(1)
7 A(1)
8 B(1)
9 C(1)

24
Answers

10 (a) mention of Doppler effect OR relative motion (d) speed of sound relative to the microphone is less;
between source and observer produces frequency/ wavelength unchanged so frequency is lower OR
wavelength change; fewer waves recorded in unit time/per second so
Accept answers which refer to a double frequency shift. frequency is lower (2)
Award [0] if there is any suggestion that the wave speed is (e) ​845 = 850 × ​_ 340 − v
​​;
changed in the process. 340
​v = 2.00 m ​s​​  ​​
−1
(2)
the reflected waves come from an approaching
“source” OR the incident waves strike an 12 (a) A; because it is the point nearest the Earth (2)
approaching “observer”; (b) C; because it is the point moving fastest toward
increased frequency received by the device or by the Earth (2)
the car (2) 13 (a) For a stationary source and observer, ​​cs​  ​​ = f λ​
(b) ALTERNATIVE 1 (i) ​​λ​ s​​= λ + ​_vf ​​, where ​f = 500​Hz
the car is a moving “observer” and then a moving
The wave velocity, ​​cs​  ​= ​λs​  ​​ ​fs​  ​​​;
“source”; so the Doppler effect occurs twice (2)  ​c​  ​​  ​c​  ​​
ALTERNATIVE 2 ​​ ​f​  ​​ s ​ = λ + ​_vf ​= _
Therefore, _ ​ f s ​ + ​_vf ​​ ;
s
the reflected radar appears to come from a ​c​  ​​  f
“virtual image” of the device; traveling at 2v ​​fs​  ​= _ ​340
​​c​  ​​  +s v ​= _ × 500
340 + v
​​ (3)
s
toward the device (2) (ii) The frequency received is obtained by
(c) For both alternatives, allow ecf to conclusion if v OR Δf replacing ​v​ by ​− v​:
are incorrectly calculated.
​340
​​fs​  ​= _ × 500
340 − v
​​
ALTERNATIVE 1
( ​ ​ × ​(9.5 × ​10​​  3​)​
8) The frequency ​​fwall
​  ​​​will be reflected and
  ​​​ 3 × ​10​​ 
v = ________________
   ​ =​36 m s–1;
( 9)
​ 40 × ​10​​  ​ ​ × 2 detected by the observer:
36 > 28 so car exceeded limit (2)
​​freflected
​  ​340
​= _ × 500
340 − v
​​ (3)
There must be a sense of a conclusion even if numbers are
not quoted. (b) ​30 = _ ​340 × 500 _
340 − v
​ − ​340 × 500
340 + v
​​ ;
ALTERNATIVE 2 340 × 500
_
​​v​​  ​ + ​ 15 ​  v − ​340​​  ​= 0​
2 2

reverse argument using speed limit.


As ​​_ v
​ ≪ 1​we can neglect; (​​​ _ v 2
​  340  ​)​​​  ​​;
​​ 2 × 40 × ​10​​ 8 ​ × 28​  =​ 7500 Hz;
9
​Δf =​ ____________
   340
3 × ​10​​  ​
​v ≈ _ ​15500
× 340
​= 10.2​m s−1(4)
9500 > 7500 so car exceeded limit (2)
There must be a sense of a conclusion even if numbers are
not quoted.
340 D.1
11 (a) wavelength​= ​_
850
​ = 0.40 m​;
path difference ​= 1.8 m​;
1.8
Exercises
​1.8 m = 4.5λ​ OR _ ​0.20 ​= 9 ​half wavelengths;
1 1.61 m s–2
waves meet in antiphase at P OR destructive
interference/superposition at P (4) 2 24.7 N kg–1
Allow approach where path length is calculated in terms 3 7.34 N kg–1
of number of wavelengths; along path A 4 6.69 × 10–8 N kg–1
(​56.25​) and path B (​60.75​) for MP2, hence path
difference ​4.5​wavelengths for MP3 5 0 N kg–1
(b) equally spaced maxima and minima; 6 30 J kg–1
a maximum at Q; four additional maxima 7 90 J
between P and Q (2)
8 240 J
(c) the amplitude of sound at Q is halved;
intensity is proportional to amplitude squared 9 0J
I​ ​  ​​ 1 10 0 J
hence _
​​ ​IA​  ​​ ​= _
​4 ​​ (2)
0
11 (a) 1.6 MJ kg–1 (b) 3.1 × 109 J

25
14 0J Do not penalize candidates if they state
15 0J Answers ‘push of the track on the marble’.
(iii) loss in PE = 0.05 × 10 × (0.8 – 0.35);
16 (a) 1.6 MJ kg–1 (b) 3.1 × 109 J
= gain in KE = 12 mv2;
(c) and (d)
(c) and (d) to give v = 3.0 1
(b) ALTERNATIVE m s–1;
V or use of universal gravitational force/acceleration/
orbital velocity equations; equating to centripetal
use of v = 2gh to give v = 4.0 m s–1 at _
x force or acceleration; rearranges to get ​k = ​ G 2 ​​ (3)
point B; 4 ​π​​  ​
ALTERNATIVE
and then use of2 v2 – u2 = 2gh with
substitution –1 proper equation for ​T​from orbital
field strength is zero v = 4.0 m sof and h = 0.35 m;
motion equations; substitution of proper equation
when gradient is zero to get u = 3.0 m s–1; [3]
for M
​ ​ OR ​R​from orbital motion equations;
2.38××10
10 ​ Gcandidate
3 3 m –1
17
12 2.38 m s s–1 Do not penalize
rearranges to get ​k the =_ ​​ if g = (3)
4 ​π​​  2​
9.8 m s ​R​  is
–2
E used.
(c) ​​M​ Mars​= (​ ​R​ Mars  ​​​ )​​​  ​ ​​(​ _​T​  ​​ ) Earth
13
18 Hydrogen
Hydrogenwouldwouldescape.
escape. ​​ 3
​TEarth
​  ​​ 2
​_  ​ ​​​  ​ ​M​  ​​​
14 3 km Earth Mars
19 3 km or other consistent re-arrangement
15 2.74 km s–1
20
16 2.74 km
graph of sT 2 vs. r 3
–1
​ 6.4 ×  ​10​​  23​ ​kg
= (2)
(d) we know that field strength and potential gradient
17 4.2 × 107 m 1 due
are equal; which means that the field strengths
18 1.5 hours to the Earth and the Moon are equal and opposite at
19 (a) 5.9 × 1010 J (b) –1.2 × 1011 J the position where the potential is maximum; at this
PH_SB_IBGLB_9021_ANS.indd 1 22/07/2014 16:25
(c) –6.1 × 1010 J position, we equate the gravitational field strength of
the Earth and Moon (3)
Challenge yourself 6 (a) B (1)
_
1 Since _
1 Mm
​​ 2 ​  m ​v​​  2​  = G ​_
radius.
√GM _
2r ​​, ​v = ​​  r ​  ​​ where ​r​is the orbital
(b) Beneath the surface, some of the Earth’s mass is
above you, which cancels out that between you
_ and the center.
We have already established that ​​vescape
​  ​​ = ​_
​  R ​  ​​, √ 2GM
(1)
Gravity

where ​R​is the radius of the body that is responsible


for the field.
Notice that these equations are dimensionally Distance
equivalent. Take care in assessments to select the Earth
correct one and to use the correct distance.

Practice questions 7 (a) B (1)


(b) The force between the Moon and the Earth (in
1 A(1) this example) remains attractive. (1)
2 D(1) 4 ​π​​  ​ 3 2
8 (a) ​​T​​  2​= _
​G ​M​  ​​
​ ​r​​  ​​ (1)
3 C(1) g

(b) ​​M​ g​= 3.4 × ​10​​  41​  kg​ (1)


4 (a) circular motion involves a changing velocity;
(c) The number of stars like the Sun ​≈ 2 × ​10​​  ​​ (1) 11
Tangential velocity is always perpendicular to
centripetal force/acceleration; there must be a In case of interest, the low marks allocation emerges from
force/acceleration toward center/star; without a this question being on an Olympiad paper.
centripetal force the planet will move in a straight 9 (a) the work done per unit mass; in bringing a (small
line(2) test) mass from infinity to the point (2)
(b) gravitational force/force of gravity (1) Idea of ratio crucial for first mark.
​GM2 ​​;
(c) use of ​g = _ (b) (i) ​g = ​_ _
G ​Mp​  ​​ G ​M​  ​​
2 ​ − ​ 2 ​​;
m
​R​​  ​
r​​ 1​  ​​​​  ​ ​​r2​  ​​​​  ​
6.4 N kg OR m s (2)
−1 −2
​Mp​  ​​ M
​ ​  ​​ ​Mp​  ​​
5 (a) the force/field and the velocity/displacement are at ​0 = ​_2 ​ − ​_ m _
​; ​ ​= 16​ (3)
0. ​8​​  ​ 0. ​2​​  2​ ​Mm​  ​​
90° to each other OR there is no change in GPE of
the moon/Phobos (1)

26
Answers

(ii) ​​E​ k​​= mΔV; ALTERNATIVE 2


= 1500 × ​(4.6 − 0.20)​ × ​10​​  7​; ​r​/ distance between the centers of the objects /
= 6.6 × ​10​​  10​  J​ (3) orbital radius remains unchanged;
GMm
Award [2] if attempted use of ΔV but value used is ​  ​= − ​_12 ​ ​_
since ​​ETotal r ​​, energy will not change (2)
wrong and [1] if an individual potential value rather 16 (a) speed to reach infinity/zero gravitational field OR
than a difference is used. speed to escape gravitational pull/effect of planet’s
10 (a) (i) Vsurface = –6.3 (± 0.3) × 107 J kg–1(1) gravity(1)
(ii) Vh is at R = 42 × 106 m; Do not allow reference to leaving/escaping an orbit.
= –1.0 (±0.2) × 107 J kg–1(2) Do not allow “escaping the atmosphere”.
Watch for R = 3.6 × 107 m being used. If so award 9 GMm
(b) kinetic energy at take-off =​ ​_
16
​ × ​_R
​​;
[1] and use ECF.
kinetic energy at take-off + gravitational potential
(b) ΔV = 5.3 (± 0.5) × 107 J kg–1; energy = gravitational potential energy at
Energy = mΔV; = 5.3 (±0.5) × 1011 J (3) maximum height OR​ ​_ 9 GMm _
​ × ​_ ​ − ​GMm GMm
​= − ​_
16 R R r ​​;
Award [2] if student calculates the EP of the satellite 9R
(1011 J). solves for r and subtracts R from answer =​ ​_7
​​ (3)
(c) Any two of the following ([1] each): Award [0] for work that assumes constant g.
the satellite has to be given a horizontal velocity (c) energy reduces/lost; radius decreases; speed
(or has to have EP) to go into orbit; rocket motors increases(3)
lifting rocket not 100% efficient; air resistance in Do not allow “kinetic energy reduces” for MP1
initial stages of launch (2)
11 (a) because the force is always at right angles to
the velocity / motion / orbit is an equipotential D.2
surface(1)
Do not accept answers based on the displacement being Exercises
zero for a full revolution.
GMm 1 2 × 10–4 N
(b) (i) equating gravitational force ​​_ ​​;
​r​​  2​
_m​v​​  2​
to centripetal force ​​ r ​​to get result (2) 2 20 N C–1, south
3 5.7 × 10–10 N C–1
​​GMm
(ii) kinetic energy is _ 2r
​​;
GMm 4 (a) 1.8 × 106 N C–1
addition to potential energy ​− ​_ r ​​ to (b) 4.5 × 105 N C–1
get result (2)
(c) 0.045 N
(c) the total energy (at the new orbit) will be greater
than before / is less negative; hence probe (d) 0.01 N
engines must be fired to produce force in the 5 (a) 1 × 10–7 N
direction of motion / positive work must be (b) 1 × 10–5 m s–2
done (on the probe) (2)
6 2.25 × 106 V
Award [1] for mention of only potential energy increasing.
7 1.13 × 106 V
12 A(1)
8 (a) Q1 positive
13 B(1)
(b) towards Q2
14 B(1)
9 F
GM _ GMm GMm
E = ​_12 ​  m ​_
15 (a) ​ _1_
r ​ − ​ r ​= − ​2 ​ ​ r ​​; 10 (a) 20 V
GM
comparison with ​V = − ​_ r ​​to give answer (2) (b) 10 V
(b) ALTERNATIVE 1 (c) 0 V
at the position of the planet the potential depends 11 (a) 40 J
only on the mass of the star /does not depend on (b) –20 J
the radius of the star; the potential will not change (c) 0 J
and so the energy will not change (2)
12 50 V m–1; field not uniform

27
Answers

13 –3 nC 8 (a) minimum of three lines equally


14 (a) –10 eV spaced and distributed, perpendicular
to the plates and downwards; e.g.
(b) –50 eV
(c) 20 eV
15 2 V E
16 5 V
17 15 J
18 4 J edge effect shown (2)
19 –8 J Condone lines that do not touch plates
20 (a) It accelerates downwards. (b) ​4.3 × ​10​​  5​ ​ N C​​  −1​​ (1)
(b) 12 J (c) ​Δ ​EP​  ​​ = qΔV = 3.2 × ​10​​  −19​  × 5.2 × ​10​​  3​​;
​= 1.7 × ​10​​  −15​  J​;
21 4 eV
negative/loss(3)
22 3 eV
9 (a) B (1)
Challenge yourself (b) The charge of the electrons that move is equal in
terms of both surplus and deficit. (1)
1 3 cm
10 (a) B (1)
Practice questions (b) The entirety of the bird is at the same potential
and so no current flows through it. (1)
1 C(1)
11 (a) Electric force = ​E ​n1​  ​​  e​;
2 C(1) ​E ​n1​  ​​  e = mg + 6πηa ​u1​  ​​​
3 A(1) ​E ​n2​  ​​  e = mg + 6πηa ​u2​  ​​​;
4 C(1) Subtracting: ​Ee​(​n2​  ​​  − ​n​ 1​​)​= 6πηa( ​u2​  ​​ − ​u1​  ​​ )​;
6πηa
5 C(1) ​​n2​  ​​ − ​n1​  ​= _
​ Ee ​ ( ​u2​  ​​ − ​u1​  ​​ )​;
V _
6 B(1) Since ​E = ​_
d
​= ​ 5000 −2 ​​ and ​u = _
​distance
time
​​ ;
1.5 × ​10​​  ​
k×q
E =_ ​6π ​ ​(​  ​142
× 1.82 × ​10​​  ​ × 2.76 × ​10​​  ​ _ 0​​  ​ _
 ​  − ​  ​178  ​)​​
0​​  ​
−5 −6 −2 −2
7 (a) ​ ​ 2 ​​; ​​n​  2​​ − ​n​  1​= ______________________
     
​r​​  ​ 5000
​(_
​  −2 ​)​ × 1.6 × ​10​​ 
−19

​8.99 × ​10​​  ​ ×2 6.0 × ​10​​  ​​​
9 −3
1.5 × ​10​​  ​
​E = _________________
  
0. ​4​​  ​ ​​n2​  ​​ − ​n1​  ​​= 1.95​;
OR ​E = 3.37 × ​10​​  8​ N ​C​​  −1​​ (2) The change in the number of electrons is a
Ignore any negative sign. decrease of 2. (8)
F = q × E = 1.6 × ​10​​  −19​  × 3.4 × ​10​​  8​
(b) ​ (b) ​ QE = m ​ 1​  ​​  g + 6πη ​r1​  ​​ ​u1​  ​​​ and ​QE = m ​ 2​  ​​  g + 6πη ​r2​  ​​ ​u2​  ​​​;
= 5.4 × ​10​​  −11​  N​; ​2QE = ​(​m1​  ​​  + ​m​ 2​​)​g + 6πη( ​r1​  ​​ ​u1​  ​​ + ​r2​  ​​ ​u2​  ​​ )​
​a = _F ​= _
​m ​5.4 × ​10​​ −31 ​ ​ = 5.9 × ​10​​  19​ m ​s​​  −2​​
−11
(2) Considering the combined drop: ​
9.1 × ​10​​  ​
2QE = m ​ 3​  ​​ g + 6πη ​r3​  ​​ ​u3​  ​​​
Ignore any negative sign.
Award [1] for a calculation leading to a
​ = < < m ​s​​  −2​> >​ Now, ​​m3​  ​​ = ​m1​  ​​  + ​m​ 2​​​ so ​​r3​  ​​ ​u3​  ​​ = ​r1​  ​​ ​u1​  ​​  + ​r​ 2​​ ​u2​  ​​​
Award [2] for bald correct answer Volume is conserved, so ​​_43 ​  π ​​r3​  ​​​​  3​= _
​43 ​  π ​​r1​  ​​​​  3​ + ​_43 ​  π ​​r2​  ​​​​  3​​
(c) the electron moves away from the point charge/ 3_

to the right along the line joining them; √


and ​​r3​  ​= ​ ​​r1​  ​​​​  3​  + ​​r2​  ​​​​  3​  ​​
​r​  ​​ ​u​  ​​  + ​r​  ​​ ​u​  ​​
decreasing acceleration; Hence, ​​u3​  ​= _
​ 31_
1 2 2
​​ (5)
increasing speed (3) √
​ ​​r1​  ​​​​  3​  + ​​r2​  ​​​​  3​ ​
Allow ECF from MP1 if a candidate mistakenly evaluates 12 D(1)
the force as attractive so concludes that the acceleration
13 A(1)
will increase
14 D(1)
15 B(1)

28
Answers

16 (a) identifies units of ​σ​ as ​C ​m​​  −2​​; V


(2)
_​​ C ​ × ​_
N ​m​​  2​
​​ seen and reduced to ​N ​C​​  −1​​ (2)
​m​​  2​ ​C​​  2​ 
(b) Horizontally, ​F =​ ​T sin 30°​ 0 R
0
Vertically, T
​ cos 30° = mg​; r

_ F ​ = tan 30°​;
​​ mg
​ = mg tan 30°= 0.025 × 9.8 × tan 30° = 0.14 N​
F (3) W 1.7 × ​10​​  −16​
ΔV = _
(c) ​ ​q ​= _
​1.60 × ​10​​  −19​​ = 1.1 × ​10​​  3​  V​ (1)
Allow g = 10 N kg−1
(d) ​= 8.99 × ​10​​  ​  × Q × ​(_ 1.0 × ​10​​  −1​)
1 1
Award [3] marks for a bald correct answer.
9
​  5.0 × ​10​​  −2 ​ ​   − ​ _  ​ ​​;
Award [1max] for an answer of zero, interpreting that Q
​ = 1.2 × ​10​​  −8​  C​ (2)
the horizontal force refers to the horizontal component of (e) to highlight similarities between different fields (1)
the net force.
​Fq ​= _
E=_
(c) ​ ​ 0.14 −6 ​ = 1.2 × ​10​​  5​​;
1.2 × ​10​​  ​
​ = 2E ​ε0​  ​= 2 × 1.2 × ​10​​  5​ × 8.85 × ​10​​  −12​
σ D.3
= 2.1 × ​10​​  −6​ C ​m​​  −2​​ (2)
Allow ECF from the calculated F in (b)(i) Exercises
Award [2] for a bald correct answer.
Q
1 (a) 2 × 10–5 N
​1.2 × ​10​​  ​ −6
(d) ​​_2 ​= _ ​​ (b) east
0. ​22​​  ​ 2
0. ​18​​  ​
Q
​ = + 1.8 × ​10​​  −6​  C​; 2 (a) 5 × 10–6 N
2 s.f. (2) (b) west
Do not award MP2 if charge is negative
3 (a) up
Any answer given to 2 s.f. scores MP2
(b) right
17 (a) ALTERNATIVE 1 (c) up
work done on moving a positive test charge in any
4 4 × 10–19 N
outward direction is negative; potential difference
is proportional to this work so ​V​decreases from A 5 (a) 8.0 × 10–17 J
to B (2) (b) 1.3 × 107 m s–1
ALTERNATIVE 2 (c) 7.4 × 10–4 T
potential gradient is directed opposite to the field 6 4 × 10–20 N
so inwards; the gradient indicates the direction of
increase of ​V​, hence V ​ ​increases toward the center/ Practice questions
decreases from A to B (2)
ALTERNATIVE 3 1 C(1)
kQ 2 D(1)
​V = ​_R
​​ so as ​r​increases ​V​ decreases;
​ ​is positive as ​Q​is positive
V (2) 3 D(1)
ALTERNATIVE 4 4 A(1)
the work done per unit charge in bringing a 5 A(1)
positive charge from infinity;
6 A(1)
to point B is less than point A (2)
(b) The sphere is solid and conducting, which means 7 A(1)
the field strength within the sphere’s radius is 8 (a) force per unit charge;
zero (and therefore potential must be constant acting on a small/test positive charge (2)
so that potential gradient is zero). Beyond the (b) horizontally to the left (1)
sphere’s radius, an inverse proportionality exists: ​
Arrow does not need to touch X
V∝_ ​1r ​​

29
Answers

(c) proton moves to the right/they move in opposite shows force is downwards by any valid rule;
directions; reading of balance increases (3)
force on each is initially the same; Allow ECF from “out of page”
proton accelerates less than electron initially ALTERNATIVE 2
because mass is greater; currents are antiparallel / in opposite directions;
field is stronger on right than left as lines closer; so wires repelled (by any argument giving force
proton acceleration increases as it is moving into direction);
stronger field OR electron acceleration decreases reading of balance increases (3)
as it is moving into weaker field (max. 4) (b) ​2.6 × ​10​​  ​ ​ N m​​  ​​
−5 −1
(1)
Allow ECF from (b) (c) volume of the wire (a cylinder) ​= π ​r​​  2​  l
Accept converse argument for electron
= π × ​​(_ )​​​  ​ × 0.15 = 7.36 × ​10​​  ​ ​m​​  ​​;
2
​  2.5 × ​10​​  ​
−3

2
 ​ −7 3

9 (a) magnetic force is to the left at the instant shown charge in the wire
OR explains a rule to determine the direction
of the magnetic force; force is perpendicular to ​= 8.5 × ​10​​  28​ × 7.36 × ​10​​  −7​ × 1.6 × ​10​​  −19​
velocity/direction of motion OR force is constant = 10 × ​10​​  3​  C​;
​BIl ​2.6 × ​10​​ −4 ​  × 0.15
−5
in magnitude; force is centripetal/toward the ​v = _Bq
​= ______________
  
   4 ​  ;
1.3 × ​10​​  ​  × ​10​​  ​
center (max. 2)
= 3.0 μm ​s​​  −1​​ (4)
NOTE: Accept reference to acceleration instead of force
(d) parts of the wire will experience a smaller
​m ​Rv​​  ​​​;
2
qvB = _
(b) ​ magnetic field; and hence a smaller force; so the
​1.67 × ​10​​  ​ × 2.0 × ​10​​  ​
​R = __________________
     
−27 6
​​ OR 0.060 m (2) reading of the balance will decrease (3)
1.6 × ​10​​  ​ × 0.35
−19

NOTE: Award MP2 for full replacement or correct 12 (a) B (1)


answer to at least 2 significant figures (b) Currents flowing in the same direction ‘pinch’
together, because the magnetic fields between
(c) ​ ​2πR
T=_ v ​  ; them are in opposite directions. (1)
​2π × 0.06
T =_ 6 ​ = 1.9 × ​10​​  ​  s​
−7
(2)
2.0 × ​10​​  ​
(d) ALTERNATIVE 1
work done by force is change in kinetic energy; D.4
work done is zero/force perpendicular to velocity(2)
NOTE: Award [2] for a reference to work done is zero Exercises
hence Ek remains constant
ALTERNATIVE 2 1 (a) 2 × 10–4 V
proton moves at constant speed; (b) 1 × 10–4 A
kinetic energy depends on speed (2) (c) 2 × 10–8 J
NOTE: Accept mention of speed or velocity indistinctly (d) 2 × 10–8 J
in MP2
(e) 20 m
10 (a) out of the page plane / ⊙(1) (f) 1 × 10–8 N
Do not accept just “up” or “outwards”
2 (a) 1 × 10–6 T m2
(b) 8.5 × 1.60 × 10–19 × 6.8 × 105 = 9.2 × 10–13 N(1)
(b) 0.25 × 10–6 T m–2 s–1
(c) the magnetic force does not do work on the
(c) 0.25 μV
electron hence does not change the electron’s
kinetic energy OR the magnetic force/ 3 (a) 1.5 × 10–5 T m2
acceleration is at right angles to velocity (1) (b) 1.3 × 10–5 T m2
(d) the velocity of the electron is at right angles to (c) 0.67 μV
the magnetic field; 4 (a) (i) 100 π rad s–1
(therefore) there is a centripetal acceleration / (ii) 3.9 V
force acting on the charge (2) (iii) 2.8 V
11 (a) ALTERNATIVE 1 (b) 1.4 V
magnetic field due to upper wire on lower wire 5 48.4 Ω
horizontal and into page;

30
Answers

Challenge yourself 5 C(1)


6 C(1)
1 When the motor coil is stationary, there is no induced
emf to oppose the current. 7 (a) The magnetic field at the position of the ring is
increasing because the magnet gets closer to the
Practice questions ring.(1)
(b) e.g. (1)
1 (a) force exerted per unit mass; on a small / point
mass(2)
(b) from the law of gravitation, the field strength S
_F ​ = G ​_
​m M ​= ​g​  ​​​
​R​​  2​ 0

to give GM = g0R2(2)
N.B. To achieve full marks, candidates need to
F ​= ​g​  ​​​
(c) since the induced magnetic field is upwards OR
state that _
​​m 0 by Lenz law the change of magnetic field/flux
(c) downwards must be opposed OR by conservation of energy
(accept 90° to B field or down the wire) (1) the movement of the magnet must be opposed;
F = Bev cos θ​
(d) ​ (1) therefore the force is repulsive/upwards (2)
(e) work done in moving an electron the length of 8 (a) long sides of coil AB/CD cut lines of flux OR flux
the wire is linkage in coil is changed;
​W = FL = BevL cos θ​; induced emf depends on rate of change of flux
emf = work done per unit charge; linked OR rate at which lines are cut;
therefore, ​E = BLv cos θ​ emfs acting in sides AB/CD add / act in same
OR direction around coil;
​electric field = _ ​ Fe ​ = Bv cos θ​; process produces an alternating/sinusoidal emf (3)
emf E = electric field × L; “Induced” is required
to give E ​ = BLv cos θ​ (3) Allow OWTTE or defined symbols if “induced emf” is
Award [2] if flux linkage argument is used. given.
Mm _ m ​v​​  ​ 2 Accept “induced” if mentioned at any stage in the context
F = G ​_
(f ) ​ 2 ​= ​ R ​​; of emf or accept the term “motional emf”.
​R​​  ​
​g​  ​​ ​R​​  2​ Award [2 max.] if there is no mention of “induced emf”
​GM
such that ​​v​​  2​= _ R
​= ​_
0
R
​​;
(b) 0.058 V (1)
​10 × ​​6.4 ​​​  ​  × ​10​​  ​​ ​
( )2 12
​​v​​  2​= _____________
   (c) 160 × 0.058 = 9.2 V (1)
6.7 × ​10​​  ​ 6

to give v = 7.8 × 103 m s–1(3) If 80 turns used in (b), give full credit for (b) × 2 here
(g) ​ ​Bv E
L=_ cos θ
​; 9 (a) A (1)
= ​_____________________
  
   ​10​​  3​
​ (b) An electromagnet forms when a current-carrying
6.3 × ​10​​  −6​ × 7.8 × ​10​​  3​ × 0.93 conductor is wrapped around a magnetic material.
= 2.2 × ​10​​  4​  m​ (2) When the conductor has no current, a stationary
2 (a) (i) emf (induced) proportional to; rate of change magnet cannot induce an emf. (1)
/cutting of (magnetic) flux (linkage); (2) 10 (a) A (1)
(ii) magnetic field / flux through coil will (b) A current is indeed induced because of the
change as the current changes (1) rotation of a conductor relative to a magnetic field. (1)
(b) (i) sinusoidal and in phase with current (1)
(ii) sinusoidal and same frequency; with 90°
phase difference to candidate’s graph for ​ϕ​ (2)
(iii) emf is reduced; because B is smaller (2)
E.1
Award [0] for ‘emf is reduced’ if argument fallacious.
(c) advantage: no direct contact with cable required; Exercises
disadvantage: distance to wire must be fixed (2) 1 (a) 17 p, 18 n
3 D(1) (b) 28 p, 30 n
4 B(1) (c) 82 p, 122 n

31
Answers

2 4.16 × 10–18 C, 9.04 × 10–26 kg (c) Bohr assumed electrons were in circular orbits
3 235 around nucleus; of fixed angular momentum;
92U
that were stable (did not radiate); and thus the
4 92 protons, 146 neutrons energy could be calculated (3)
5 (a) 1.9 × 107 m s–1 2 (a) Mark both processes, 1 and 2, together.
(b) 4.9 × 10–15 m Award [1] for any two of the following:
6 10 collisions with (external) particles; heating the gas
7 13.06 eV, 3.15 × 1015 Hz to a high temperature; absorption of photons (2)
8 0.31 eV, 7.44 × 1013 Hz E=_
(b) (i) ​ ​hc
λ

6.63 × ​10​​  −34​  × 3 × ​10​​  8​
9 13.6 eV, 3.28 × 1015 Hz = ________________
 ​   
   ​;
658 × ​10​​  ​
−9

10 5.3 × 10–11 m = 1.89 eV​ (2)


11 2.5 × 1015 Hz (ii) electrons absorb photons (of energy 1.89
eV) to make a transition from n = 2 to
Practice questions n = 3; on de-excitation, photons of energy
1.89 eV, i.e. wavelength 658 nm are emitted;
1 (a) (i) Answer to include: in all directions, however, and not just
missing frequencies / wavelengths; along the initial direction, hence intensity is
in otherwise continuous spectrum (2) reduced(3)
(ii) Answer to include: 3 B(1)
light from Sun is split into its component 4 C(1)
wavelengths; using prism / grating (2)
(b) (i) correct substitution into E = hf and 5 A(1)
​hc
c = f λ to give ​E = _λ
​​; 6 A(1)
7 D(1)
​6.63  
× ​10​​  ​  × 3 × ​10​​  ​
–34 8
​E = ________________
   ​ = 3.38 × ​10​​  –19​  J​ (2)
5.88 × ​10​​  ​
–7

(ii) transition is an absorption so involves 8 C(1)


electron being ‘promoted’ up between two 9 B(1)
levels; energy of gap must be exactly = 3.38 10 D(1)
× 10–19 J; this is between
(–5.80 × 10–19 J) and (–2.42 × 10–19 J) levels; (3) 11 (a) most undeflected/pass straight through;
[2] can be given for other relevant information hence mostly empty space;
concerning, for example, the existence of photons few deflected (allow “bent”, “reflect”, “bounce back” etc);
with different energies in sunlight / the immediate hence small dense nucleus
reradiation in random directions. The final mark is positive / positively charged; (max. 4)
for identifying
This the energy
can also just be shownlevelsonconcerned.
a = 0.001 297 5 × 10
(b) electron –26
kg ;
accelerated / mention of centripetal force;
diagram.
This can also just be shown on a diagram. correct substitution intoEM
should radiate E = waves/energy;
mc2;
energy/10–19 J
energy released
and spiral into the nucleus (3)
0 = 0.001 297 5 × 10–26 × 9.00 × 1016 J
12= 1.17 10–12 J; λ = 435 nm;
(a) ×identifies [3]
​hc ​6.63  
× ​10​​  ​  × 3 × ​10​​  ​
−34 8
​E = _
(f) (i) an (electron) ​= ________________
  
λantineutrino;
​​ [1]
;
–1.59 4.35 × ​10​​  −7​
−19
–2.42
4.6 ×10 J
Reject ‘neutrino’. (3)
–3.00 (b) −0.605 OR −0.870 OR
(ii) idea that there is a fixed total energy of −1.36 to −5.44 AND
decay; arrow pointing downwards (1)
Arrow
total energy MUST
shared match calculation
between the (three) in (a)
(c) Difference
resulting in energy levels is[2]equal to the energy
particles / OWTTE;
–5.80
of the photon;
(iii) correct calculation of decay constant λ;
λ= ln 2Downward
= 0.845
arrow as energy is lost by hydrogen/
–7.64 0.82energy is given out in the photon/the electron falls
correct substitution into N = N0e–λt;
from a higher energy level to a lower one (2)
(c) Mark (i) and (ii) together. [1] for each to give N = N0e–8.45 therefore N = e–8.45
relevant point e.g. N0
= 0.000 213 = 0.02%; [3]
Bohr assumed electrons were in circular
N.B. Award attempts without full
orbits around nucleus;
equation [1].
32
of fixed angular momentum;
Answers

13 (a) equating centripetal to electrical force (b) 1.6 × 10–17 J


_ 2k​e​​  2​ m​v​​  2​
​​ ​r​​  2​ ​= _
​ r ​​to get result (1) (c) 1.2 × 10–10 m
k​e​​  2​ 2k​e​​  2​ 6 4.4 × 10–38 m; the opening is too small
(b) uses (a) to state ​​Ek​  ​= _ ​ r ​​ OR states ​​Ep​  ​= − ​_r ​​;
k​e​​  ​ 2k​e​​  ​
2 2 7 54°
adds ​​ET​  OT​= ​Ek​  ​​ + ​Ep​  ​= _
​ r ​ − ​_ r ​​to get the result (2)
(c) the total energy decreases OR by reference to ​​ Practice questions
k​e​​  2​
E​ TOT​= − ​_ r ​​; the radius must also decrease (2)
1 (a) aspect:
NOTE: Award [0] for an answer concluding that radius
electrons will not be emitted unless the frequency
increases
of light exceeds a certain minimum value /
(d) with ​ n = 3, v = 1.44 × ​10​​  6​  m ​s​​  −1​​; electrons are emitted almost instantaneously with
​λ = 5.05 × ​10​​  −10​  m​ (2) the light falling on the surface even if light is of
2πr 2π × 9 × 2.7 × ​10​​  −11​
(e) ​​_ _______________​ = 2.99 ≅ 3​ 
λ ​= ​   5.1 × ​10​​  −10​
(1) very low intensity / the energy of the electrons
emitted is not affected by the intensity of light
14 (a) the electrons accelerate and so radiate energy;
falling on the surface (1)
they would therefore spiral into the nucleus/
corresponding explanation:
atoms would be unstable; electrons have discrete/
only certain energy levels; the only orbits where light consists of photons whose energy is hf
electrons do not radiate are those that satisfy the hence no electrons are emitted unless hf is larger
h than the energy needed to escape the metal / an
Bohr condition​ mvr = n ​_

​​ (max. 3)
electron is emitted as soon as it absorbs a photon.
​m​  ​​ ​v​​  2​
k​e​​  ​ 2
1 1 1_ k​e​​  ​
2
If the photon has sufficient energy no delay is
(b) ​​_ _ _ _ 2 _
r ​= ​ 2 ​  ​OR ​Ek​  ​​​= ​​ 2 ​ ​Ep​  ​​​hence ​​ 2 ​​mev = ​​ 2 ​ ​ r ​​
e
​r​​  ​ required / the intensity of light plays no role in the
solving for v to get answer (1)
energy of the electron, only the frequency of light
Answer given – look for correct working does(1)
_


​  k​e​  ​​  ​​ r​​  ​​ with ​​me​  ​​ vr = _ h
2
(c) combining ​v = ​_​m ​2π ​​ using correct (b) (i) the threshold frequency is found from the
e
2_k​e​​  ​ 2 _
2
​h​​  ​
2 frequency axis intercept;
substitution, e.g. ​​​me​  ​​​​  ​ ​​m​  ​​  r ​ ​r​​  ​= ​ 2 ​​;
e 4 ​π​​  ​ to be 3.8 (±0.2) × 1014 Hz (2)
correct algebraic manipulation to gain the answer (2) (ii) a value of the Planck constant is obtained
Answer given – look for correct working from the slope;
Do not allow a bald statement of the answer for MP2. to be 6.5 (±0.2) × 10–34 J s (2)
Some further working e.g. cancellation of m or r must be Award [0] for ‘bald’ answer of 6.63 × 10 J s.
–34
shown
(iii) the work function of the surface is found
(d) r = 5.3 × 10−11 m(1) from the intercept with the vertical axis;
to be 1.5 (±0.1) eV (2)
(c) straight line parallel to the first; intersecting the
E.2 frequency axis at 8.0 × 1014 Hz (2)
2 (a) all particles have a wavelength associated with
Exercises them / OWTTE;
1 (a) 9.6 × 10–20 J the de Broglie hypothesis gives the associated
(b) 7.1 × 1014 Hz wavelength as ​λ = _​ph ​​;
(c) 3.7 × 10–19 J where h is the Planck constant and p is the
(d) 5.6 × 1014 Hz momentum of the particle (3)
If answers just quote the formula from the data book then
2 (a) 8.6 eV
award [1] for showing at least students recognize which
(b) 4.3 eV formula relates to the hypothesis.
(c) 4.3 V (b) (i) Ek = Ve = 850 × 1.6 × 10–19 J
(d) 1.0 × 1015 Hz = 1.4 × 10–16 J (1)
3 no ​p​​  2​ _
(ii) use ​E = _ ​ = √​ 2mE ​​;
​2m ​​to get p
________________________
4 1.5 × 1015 Hz
substitute ​p = ​√2   
× 9.1 × ​10​​  −31​ × 1.4 × ​10​​  −16​  ​​
5 (a) 100 eV –23
= 1.6 × 10 N s (2)

33
Answers

h
(iii) ​λ = _
​p ​​; ​​ mass ​​ independent of ​A​ hence density
the ratio _
volume
6.6 × ​10​​  −34​ the same for all nuclei(2)
substitute ​λ = _
​1.6 × ​10​​  −23​​​
= 4.1 × 10–11 m (2) 12 (a) A (1)
(b) Lots of answers available, e.g.
3 A(1)
Light microscopes cannot be used for imaging
4 C(1) individual atoms; It is possible to make smooth
5 B(1) enough surfaces that good reflections can form
 (max. 1)
6 C(1)
7 C(1) This solid surface is smooth to light
waves larger than atoms.
8 C(1)
9 D(1)
10 (a) low intensity light would transfer energy to the
electron at a low rate/slowly; time would be
required for the electron to absorb the required 13 (3)
energy to escape/be emitted (2) h
−c ( f − ∆f )
(b) in the photon theory of light the electron MeV
interacts with a single photon; and absorbs
all the energy OR and can leave the metal θ
immediately(2) h
−c f
NOTE: Reference to photon-electron collision scores MP1
hc One mark for each correctly labeled side. Deduct 1 mark if not
ϕ = ​_
(c) ​ λ
​ − ​Ek​  ​​​;
closed.
​​Ek​  ​​= 1.5 eV​;
(b) ​​​(​me​  ​​ v)​​​  2​  = (​​ _
​  hc ​  f)​​​  ​ + ​(​ _
​  hc (​​ f − Δf)​)​​​  ​ − 2 ​(​ _
​  hc )​ ​​​  ​  f​(f − Δf)​cos θ​ (4)
2 2 2

​ϕ = 1.1 eV​ (3)


NOTE: Allow reading from the graph of ​Ek​  ​​ = 1.4 ​leading One mark for each term.
to an answer of 1.2 eV. (c) ​ ​12 ​ ​me​  ​​ ​v​​  2​  + h​(f − Δf)​​
hf = _ (2)
(d) similar curve lower than original; One mark for each side of the equation.
with same horizontal intercept (d) From the energy conservation equation:
e.g. I/nA
2.5 (2) ​​me​  ​​ ​v​​  2​= 2hΔf​;
2.0 Substituting into the equation from (b):
1.5
​2h ​me​  ​​ Δf
= ​(_
​  hc ​  f)​​​  ​ + ​​(_
​  hc (​​ f − Δf )​)​​​  ​ − 2 ​​(_
​  hc )​ ​​​  ​  f ​( f − Δf )​cos θ​
2 2 2
1.0

0.5
​2h ​me​  ​​ Δf
= ​(_
​  hc )​ ​​​  ​ ​f​​  2​ + ​​(_
​  hc )​ ​​​  ​​(​f​​  2​ − 2fΔf + Δ ​f​​  2 ​)​ − 2 ​(​ _
​  hc )​ ​​​  ​ ​f​​  2​  cos θ
−2.0 −1.5 −1.0 −0.5 0 0.5 2 2 2
V/V
−34 8
6.63 × ​10​​  ​  × 3 × ​10​​  ​
λ = __________________
11 (a) ​   
  
​1.60 ​​ ​ = 2.96 × ​10​​  −15​  m​
× ​10​​  −19​ × 4.2 × ​10​​  8​
(1) + 2 ​​(_
​  hc )​ ​​​  ​  fΔf cos θ​
2

(b) the shape of the graph suggests that electrons 2 ​m​  ​​ ​c​​  2​
_
​​ he ​  Δf = f​ ​​  2​ + ​f​​  2​ − 2fΔf + Δ ​f​​  2​ − 2 ​f ​​  2​  cos θ
undergo diffraction with carbon nuclei;
+ 2fΔf cos θ​
only waves diffract (2)
2.96 × ​10​​  ​ −15 2 ​m​  ​​ ​c​​  2​
(c) ​sin ​θ0​  ​= _
​4.94 × ​10​​  −15​​ = 0.599​; _
​​ he ​  Δf = ​f​​  2​​(2 − 2 cos θ)​  + Δ ​f​​  2​  − fΔf​(2 − 2 cos θ)​​
​37°​ OR ​0.64 / 0.65 rad​ (2) _ ​m​  ​​ ​c​​  2​ Δ ​f​​  2​
​​ he ​  Δf = ​f​​  2​​(1 − 1 cos θ)​ + ​_ 2
​  − fΔf​(1 − 1 cos θ)​​;
(d) the de Broglie wavelength of electrons is much
​m​  ​​ ​c​​  ​ Δf
2
Δ ​f​​  2​
longer than the size of a nucleus; Dividing by ​​f​​  2​​: _ ​​ he ​ ​ _2 ​= 1 − 1 cos θ + ​_2 ​ 
​f​​  ​ 2 ​f​​  ​
hence electrons would not undergo diffraction Δf
OR no diffraction pattern would be observed (2) − ​_ f
(​​ 1 − 1 cos θ)​​;

(e) volume of a nucleus proportional to Δ ​f​​  2​ Δf


Since ​Δf ≪ f​, ​​ _2 ​≈ 0​and ​​_ f
​ ≈ 0​.
1 3 2 ​f​​  ​
​​​(​A​​  ​ 3 ​​)​​​  ​ = A​ AND mass proportional to ​A​;
_

34
Answers

​m​  ​​ ​c​​  2​ Δf
Therefore, _
​​ he ​ ​ _2 ​= 1 − 1 cos θ​ 7 12.5 g
​f​​  ​
h ​f​​  ​
2
8 12.5 s–1
​ f = ​_2 (​​ 1 − 1 cos θ)​​
Δ (7)
​me​  ​​ ​c​​  ​
(e) (i) (1) 9 24 000 years
∆f
10 7.45 Bq
11 28.6 years
12 (a) 1.66 × 108 s
(b) 4.17 × 10–9 s –1
f
(c) 1.0 × 1022
(ii) ∆f (1) (d) 4.17 × 1013 s –1
2
(e) 1.2 × 10–12 g
1
Challenge yourself
17 4.16 × 10
0 –18 C, 9.04 × 10–26 kg 1 For small nuclei, the range of the strong force
0 θ
(~10–15 m) extends across all nucleons, which means
h ​f​​  ​ 2
U ​  ​= _
18 (iii) ​
92 Δ ​fmax
235
​ 2 ​​; every additional nucleon (irrespective of whether a
​me​  ​​ ​c​​  ​
proton or neutron) has a net attraction on every existing
19 when ​θ =146
92 protons, π​ neutrons (2) nucleon. Therefore, the nucleus increases dramatically
20 (a) 1.9 × 107 m s–1
in stability and accordingly releases significant amounts
of energy with every additional nucleon.
E.3 (b) 4.9 × 10–15 m For large nuclei, the diameter of the nucleus is greater
21 (a) 92 p, 141 n than the range of the strong force. Nucleons are attracted
Exercises to nearby nucleons due to the strong force, but protons
(b) 234.9405 u exert a slight electrostatic repulsion on distant protons
1 (a) 92
(c)p, 141 n
1.901u across the nucleus and distant neutrons exert no forces.
(b) 234.9405
(d) 1771uMeV The addition or subtraction of individual nucleons
(c) 1.901 u therefore has a limited effect on overall stability and the
(e) 7.60 MeV energy released or required in neutron changes is smaller.
(d) 1771 MeV
22
(e) 7.60 MeV 2 2.5 × 1012 J
2 10 3 1.82 MeV
binding energy per nucleon (MeV)

4 If the probability of decay is large (i.e. approaching


100% in a given time interval), then almost all of the
8
nuclei would decay in the time interval (and neither
the number of undecayed nuclei nor activity would
6 follow an exponential decay relationship). Similarly, if
the time interval is large, then almost all of the nuclei
ought to have decayed in that time.
4
Practice questions
2 1 (a) Deduct [1] for each error or omission, stop at zero.
Property Effect on rate of decay  (2)

0 Increase Decrease Stays the


0 50 100 150 200 250 same
A Temperature of

3 8.95 MeV sample
23 8.95 MeV
4 Mass of At is bigger than Po so no energy released. Pressure on

24 Mass of At is bigger than Po so no energy sample
5 2.32 MeV
released. Amount of
6 1.0 × 1019 Hz sample ✓
25 2.32 MeV

26 1.0 × 1019 Hz 35
Answers

(b) (i) ​​42​  He / ​​​ ​42​  α​​​​ 4 B(1)


​​222
86
​ ​Rn; ​​  (2) 5 C(1)
(ii) mass defect = 5.2 × 10–3 u; energy = mc2 6 A(1)
 ​5.2 × ​10​​  ​ × 1.6611u
× ​10​​  ​ × 9.00 × ​10​​  ​
−3 −27 16
​= _____________________________
       ​​ 7 A(1)
= 930 MeV;
8 A(1)
= 4.86 MeV = 7.78 × 10–13 J (3)
9 B(1)
(c) (i) (linear) momentum must be conserved;
momentum before reaction is zero; 10 A(1)
so equal and opposite after (to maintain zero 11 B(1)
total)(3)
12 D(1)
(ii) 0 = mαvα + mRnvRn;
13 D(1)
​​ ​V​  α ​​ ​= − ​(​ _​mα​   ​​​ )​; = − ​ 4 ​ = − 55.5​
_ ​V​  ​​ ​mRn
​  ​​ _222
(3)
Rn 14 A(1)
Ignore absence of minus sign. 15 A(1)
(iii) kinetic energy of α particle ​= ​_12 ​ ​mα​  ​​ ​​vα​  ​​​​  2​​;
16 D(1)
kinetic energy of radon nucleus ​= ​_12 ​​(_ 4 )
​  222
 ​ ​ ​ 17 A(1)
v
m​ α​​ ​(  ​)​​​  ​​
​ ​  ​​ 2
18 C(1)
​_ α
​  55.5
_ 1 19 A(1)
this is ​​55.5 ​​ of kinetic energy of α particle; (3)
Accept alternative approaches. 20 D(1)

2 (a) an (electron) antineutrino; (1) 21 C(1)


Reject ‘neutrino’. 22 (a) X: 26 and Y: 12;
(b) idea that there is a fixed total energy of decay; Z: v/neutrino(2)
total energy shared between the (three) Do not allow the antineutrino.
resulting particles / OWTTE(2) (b) total energy released is fixed;
(c) correct calculation of decay constant λ; neutrino carries some of this energy (leaving the
ln 2
​λ = _
​0.82 ​= 0.845​ beta particle with a range of energies) (2)
correct substitution into N = N0e–λt; (c) the time taken for half the radioactive nuclides to
N decay / the time taken for the activity to decrease
to give N = N0e–8.45 therefore ​​_
​N​  ​​
​​ = e–8.45 to half its initial value (1)
0

= 0.000 213 = 0.02% (3) Do not allow reference to change in weight.


N.B. Award attempts without full equation [1]. 23 (a) energy/mass difference = 8.450 – 8.398
3 (a) ​​ ​  K​​​ → ​ ​  Ar​​​  + ​β​​  ​​ ​e​​  ​ ​  + ν;
40
19
40
18
+( +) = 0.052 MeV;
+1​β​​  ​ ​ / +1e​​  ​​
0 + 0 +
(2) Q = 1.7 OR 1.66 OR 1.664 MeV
(b) 8.2 × 10−6 g (1) OR 2.66 × 10−13 J (2)
(b) 11–12 days (1)
λ = ___
(c) (i) ​ ​ 0.69 ​= 5.3 × ​10​​  −10​ ​year​​  −1​​
​ln​T​  2​​ ​; =_ (2)
1.3 × ​10​​  9​
​  1 ​
__
2 24 (a) ​​42​  α​​​  ​OR 42​ ​  He​​​​;
​1λ ​  ln​(_
​  N0 ​)​​;
N
​ ​  ​​
(ii) from ​N = ​N0​  ​​ e − λt, t = _ ​​222
86​ ​Rn​​​​ (2)
= 1.9 × 109 × ln (6.8) = 3.6 × 109 years (2) These must be seen on the right-hand side of the equation.
OR (b) ALTERNATIVE 1
n

8.2 ( 2 )
6 days is 5.18 × 105 s;
​​ 1.2​= ​ _
_ ​  1 ​ ​​​  ​​
activity after 6 days is ​​A0​  ​​ ​e​​  −1.4×​10​​  ​×5.8×​10​​  ​​≈ ​A0​  ​​​ OR A
−11 5

n = 2.77; = 0.9999927 A0 OR 0.9999927 ​λN ​ 0 OR states that


age = 2.77 × 1.3 × 109 _ A
index of e is so small that ​​​A​  ​​ ​​ is ≈ 1 OR
= 3.6 × 109 years (2) A – A0 ≈ 10−15 s−1(2) 0

36
Answers

ALTERNATIVE 2 26 (a) C (1)


shows half-life of the order of 1011 s or (b) The ‘mortality’ of people and radioactive nuclei
5.0 × 1010 s; are very different over time. The probability of
converts this to year (1600 years) or days and a human surviving from one year to the next
states half-life much longer than experiment decreases, whereas the probability of a nucleus
compared to experiment (2) remaining undecayed remains constant. (1)
Award [1 max.] if calculations/substitutions have people
numerical slips but would lead to correct deduction, 1000
radioactive
e.g. failure to convert 6 days to seconds but correct nuclei
substitution into equation will give MP2. 500
Allow working in days, but for MP1 must see conversion
of ​λ​or half-life to day−1. 0
0 68 years 136 years
(c) ALTERNATIVE 1
use of A = ​λ​N0; 27 (a) ​​_12 ​​ (1)
conversion to number of molecules = nNA = 3.7 × (b) Probability of no decay = _ ​​12 ​ × ​_12 ​ × ​_12 ​​;
1020 OR initial activity = 5.2 × 109 s−1;
which is ​​_18 ​​;
number emitted = (6 × 24 × 3600) × 1.4 × 10−11 ×
3.7 × 1020 OR 2.7 × 1015 alpha particles (3) ​​78 ​​
Probability of decay = _ (3)
ALTERNATIVE 2 28 (a) 4 (1)
use of N = N0​​e​​  −λt​​; (b) Tabulation of working:
N0 = n × NA = 3.7 × 1020; 1935 1 cm
alpha particles emitted = number of atoms 1936 4 cm
disintegrated = N – N0 = N0​(​ 1 − ​e​​  −λ×6×24×3600)​ ​​ 1937 42 cm
OR 2.7 × 1015 alpha particles (3)
1938 43 cm
Must see correct substitution or answer to 2+ s.f. for MP3
1939 44 cm
(d) alpha particles highly ionizing OR alpha
1940 45 cm;
particles have a low penetration power OR
1024 cm (2)
thin glass increases probability of alpha crossing
glass OR decreases probability of alpha striking (c) ​1 × ​10​​  ​​ cm OR ​1 × ​10​​  ​​ m
3 1
(1)
atom/nucleus/molecule(1) (d) 1941 4 × 10 = 40 m
Do not allow reference to tunneling. 1942 4 × 40 = 160 m;
(e) conversion of temperature to 291 K; 1943 4 × 160 = 640 m (2)
​​ 291 −5 ​​ OR
p = 4.5 × 10−9 × 8.31 × _ (e) The speed of the front page is 4n × 10 ​÷​6 months;
1.3 × ​10​​  ​
​ ​4​​  ​ × 10 ​​;
n

​​ 291 −5 ​​ ;
p = 2.7 × 1015 × 1.38 × 10−23 × _ ​3 × ​10​​  8​= ____________
  
365
_
1.3 × ​10​​  ​ ​  2 ​ × 24 × 3600
0.83 OR 0.84 Pa (3) ​4.73 × ​10​​  14​= ​4​​  n​​
25 (a) B (1) 14.67 = n log 4;
(b) Suppose Supply 2 starts with a power of ‘1’. n = 24.4 so the year will be 1964 (4)
The amount of power will half each year as
shown below. Supply 1 starts with twice as
much power: ‘2’. Its power halves twice per year
as shown below. From the beginning of Year 2
E.4
onwards, you are better off with Supply 2.
Year 0 1 2 3
Exercises
​   ​​ ​  ​​ ​   ​​ 1 10, 133.9 MeV
Supply 1 2 _ ​1 _ ​1 1
​_
2 8 32 2 135.8 MeV
​   ​​ ​   ​​ ​   ​​
Supply 2 1 ​_1 _ ​1 _ ​1 3 (a) ​​ 142
56
​​  57​ ​La​​​​ + β– +ν¯
​ ​Ba​​​​ ➞ 142
2 4 8
(b) 9 years

37
Answers

4 (a) 7 (c) (i) the fuel rods contain a lot more 238U than 235U;
(b) ​​  ​ ​Pu​​​​ ➞ 136
239
94
​​ ​  Zr​​​​ + 7n0
​​  54​ ​Xe​​​​ + 96
40
neutron capture is more likely in 238U than
235
(c) 164 MeV U with high-energy neutrons; but if the
(d) 239 g neutrons are slowed they are more likely to
produce fission in 235U than neutron capture
(e) 2.5 × 1024 atoms
in 238U(3)
(f) 4.13 × 1026 MeV
The argument is a little tricky so be generous. The
(g) 6.6 × 1013 J
candidate needs to know about there being two
5 2.7 × 1012 J isotopes present in the fuel and something about
6 (a) 3.6 × 1010 J the dependence of the fission and capture in the two
(b) 13 g isotopes on neutron energy.
(ii) control the rate at which the reactions take
Practice questions place; by absorbing neutrons (2)
(d) Look for four of the following main points and award
1 (a) (i) fission: [1] each.
nucleus splits; into two parts of similar mass energy lost by the slowing of the neutrons and
radioactive decay: fission elements heats the pile; this heat extracted
nucleus emits; by the molten sodium / pressurized water / other
a particle of small mass or a photon, suitable substance; which is pumped to a heat
or both (4) exchanger; water is pumped through the heat
(ii) ​​ 235
92

​  U ​​​  + ​1
0
​  n;​​​ → ​
90
38
​  S r​​​  + ​ 142
54
​  ​
X e​​​  + 4 ​1
0
​  n;
​​​​  (2) exchanger and turned to steam; the steam drives a
Allow ECF for RHS if LHS is incorrect. turbine; which is used to rotate coils (or magnets)
(iii) mass number unchanged; atomic number placed in a magnetic field (or close to coils) which
increases by +1 (2) produces electrical energy (4)
​p​​  2​ Alternatively, award [4] for a good answer, [2] for a fair
(b) (i) use of kinetic energy = _
​​2m ​​ / equivalent; answer and [1] for a weak answer.
correct conversion of MeV to joule 3 A(1)
(1.63 × 10–11 J);
4 B(1)
correct conversion of mass to kilogram (1.50
× 10–25 kg); 5 B(1)
momentum = 2.2 × 10–18 N s (4) 6 D(1)
(ii) total momentum after fission must be zero; 7 B(1)
must consider momentum of neutrons (and
photons)(2) 8 (a) energy required to completely separate the
nucleons OR energy released when a nucleus is
(iii) xenon not opposite to strontium but
formed from its constituent nucleons (1)
deviation < 30°);
Allow protons AND neutrons.
arrow shorter / longer (2)
–13 (b) the values in SI units would be very small (1)
(c) (i) energy = 0.25 × 198 × 1.6 × 10 ;
(c) ​140 × 8.29 + 94 × 8.59 − 235 × 7.59​OR ​184 MeV​ (1)
= 7.9 × 10–12 J (2)
(d) see ​energy = 180 × ​10​​  6​  × 1.60 × ​10​​  −19​​
(ii) use of ΔQ = mcΔT;
AND ​mass = 235 × 1.66 × ​10​​ −27​​;
energy = 0.25 × 4200 × 80; = 8.4 × 104 J (3)
​7.4 × ​10​​  13​  J ​kg​​  −1​​ (2)
( 4)
(iii) number of fissions = _
(
​​ ​ 8.4 × ​10​​ –12)​ ​​​ ;= 1.1 × 1016  ​1.2 × ​10​​ 0.36
   ​ × 24 × 3600
9
(e) energy produced in one day​ = ________________ ​
​ 7.9 × ​10​​  ​ ​
mass = 1.1 × 1016 × 3.9 × 10–25; = 4.1 × 10–9 kg (4) = 2.9 × ​10​​  ​  J​;
14

2 (a) (i) fission (1)  ​ 2.9 × ​10​​ 13 ​ ​ = 3.9 kg​


mass​  = _
14
(2)
(ii) kinetic energy (1) 7.4 × ​10​​  ​
(f ) specific energy of uranium is much greater than
(b) the two neutrons can cause fission in two more that of coal, hence more energy can be produced
uranium nuclei producing four neutrons so from the same mass of fuel / per ​kg​OR less fuel
producing eight etc.; OWTTE(1) can be used to create the same amount of energy (1)
(g) ​39​ (1)

38
Answers

(h) 75 s (1) (b) (i) atomic number: 6; mass number: 12 (2)
(i) ALTERNATIVE 1 N.B. if 6 and 12 are reversed, [1].
​10 min = 8 ​t1/2
​  ​​​ (ii) mass before = 3 × (6.648 325 × 10–27 kg)
= 1.994 497 5 × 10–26 kg
mass remaining​= 1.0 × ​​(_
​  12 ​)​​​  ​= 3.9 × ​10​​  −3​  kg​
8
(2)
mass of carbon = 1.993 200 0 × 10–26 kg
ALTERNATIVE 2 so mass defect
ln2
decay constant​  = ​ _75
 ​ = 9.24 × ​10​​  −3​ ​s​​  −1​​; = 1.994 497 5 × 10–26 – 1.993 200 0 × 10–26 kg
mass remaining​ = 1.0 × ​e​​  −9.24×​10​​ 
−3

​×600 = 0.001 297 5 × 10–26 kg ;
= 3.9 × ​10​​  −3​  kg​ (2) correct substitution into E = mc2;
energy released
= 0.001 297 5 × 10–26 × 9.00 × 1016 J
E.5 = 1.17 × 10–12 J (3)
2 (a) (i) two (light) nuclei; combine to form a more
Exercises massive nucleus; with the release of energy /
with greater total binding energy (3)
1 (a) 3.27 MeV (ii) high temperature means high kinetic energy
(b) 4.03 MeV for nuclei; so can overcome (electrostatic)
(c) 18.4 MeV repulsion (between nuclei); to come close
2 4.2 light years together / collide; high pressure so that there
are many nuclei (per unit volume); so that
3 8 min 20 s chance of two nuclei coming close together
4 1.5 × 105 years is greater (5)
5 1.3 pc, 1.56 arcsec 3 (a) (i) a proton or a neutron (1)
6 40 pc Both needed to receive [1].
(ii) the difference between the mass of the
7 3.18 μm; too small to measure on photograph
nucleus and the sum of the masses of its
8 Betelgeuse 1 (0.4) individual nucleons / the energy required
Meissa 4 (3.5) to separate a nucleus into its component
(b)
Bellatrix 2 (1.64) nucleons / OWTTE(1)
Alnilam 3 (1.7) (b) 9
E (MeV)

F
Alnitak 3 (2) U
8
Mintaka 3 (2.23) 7 (b
Saiph 2 (2.09) 6
(actual magnitudes in brackets)
5
9 (a) 1.36 × 103 W m–2 4
(b) 3.2 × 10–10 W m–2
3
10 (a) 1.2 × 10–7 W m–2
2
(b) 7.9 × 10–9 W m–2 H
1
11 5.6 × 103 light years
0
12 4.2 × 1030 W 0 25 50 75 100 125 150 175 200 225 250
A
13 (a) 7.25 × 103 K
Don’t expect Don’t
precision forprecision
expect any of these.
for any of these. 6 (a
(b) 1.6 × 108 W m–2
(i) F: between 8 and 9
(i) F: between 8 and 9; [1] (1)
Practice questions (ii) H: between 1 and 2
(ii) H: between 1 and 2; [1]
(1)
(iii) U: between 7 and 8 (1)
(iii) U: between 7 and 8; [1]
1 (a) a fusion reaction; since hydrogen nuclei are (c) general overall shape; max at F = 56, end point U (2)
joining to create helium / any other relevant (c) general overall shape;
(b
further detail / explanation (2) max at F = 56, end point U; [2]
(d) mass of nucleons
= (2 × 1.00728) + 1.00867
= 3.02323 u;
mass difference = 0.0072 u = 6.7 MeV; 39
Answers

LS = ( σ AS [TS] 4 =) σ [rS] 2 [TS] 4 ;


LV σ AV [TV] σ [rV] [TV] 4 2 4
(d) mass of nucleons = (2 × 1.00728) + 1.00867 9 (a) _ _ _
= 3.02323 u;
[rV]
2
1.54 × 10 28 _
_ 9600 4
mass difference = 0.0072 u = 6.7 MeV; = ×_;
3.85 × 10 26 [rS] 2 5800 4
6.7 _______________
binding energy per nucleon =_
rV = ( _ 3.85 × 10 26 9600 4 S )
= 2.2 MeV (3)
(e) (i) fusion
3
(1)
1.54 × 10 28 _
× √
5800 4
r = 2.3 rS
Do not award third marking point if radius of the
(3)
(ii) from the position on the graph, the energy
required to assemble two nuclei of 21 H is Sun is lost.
greater than that to assemble one nucleus (b) obtain the spectrum of the star;
of 32 He; hence if two nuclei of 21 H combine measure the position of the wavelength
to form one nucleus of 32 He energy must be corresponding to maximum intensity;
released / OWTTE (2) use Wien’s law (to determine temperature) (allow
4 C (1) quotation of Wien’s equation if symbols defined) (3)
Award [3 max.] for referring to identification of
5 B (1)
temperature via different ionizations of different elements.
6 (a) the star is (much) closer than the other star (and
10 (a) the letter S should be in the region of the
close enough to Earth) / parallax effect has been
shaded area
observed (1) 1 000 000
(1)
(b) (1)
θ 10 000

luminosity/Lo
100
D
1

1
d 100
1
Award [1] if all three (d, D, θ) are shown correctly. 10000 25 000 10 000 6000 3000
effective temperature/K
Do not allow d shown as the radius.
Accept D as a line from Earth to the star. (b) the fusion of hydrogen in the core eventually
d d d stops OR core contracts; the hydrogen in a layer
(c) sin _2θ = _
2D
OR tan _2θ = _
2D
OR θ = _
D
; around the core will begin to fuse; Sun expands
consistent explanation, eg: small angle of AND the surface cools; helium fusion begins in
d
approximation yields θ = _
D
; (2) the core; Sun becomes more luminous/brighter
Allow ECF from (b)(i), eg: if d shown as radius. (max. 3)
(d) any angular unit quoted for θ and any linear unit Ignore any mention of the evolution past the red giant stage
LW _ [rW]
2
quoted for D (1) (c) _ = 10 4 = _
−4
3000 4
_
L 2 × 4;
10 [rR] 10 000
(a) d = 275 pc
R
7 (1) ____________


r_ 10
_ −4
_3000 4
rR = 10 4 × 10 000 4 = 9.0 × 10 (2)
W −6
(b) because of the difficulty of measuring very small
angles (1) 11 (a) photon/fusion/radiation force/pressure balances
8 (a) made of dust and/or gas OR formed from gravitational force/pressure; gives both directions
supernova OR can form new stars OR some correctly (outwards radiation, inwards gravity) (2)
radiate light from enclosed stars OR some absorb (b) LGacrux = 5.67 × 10 −8 × 4π × (58.5 × 10 9) 2 × 3600 4;
light from distant stars (1) LGacrux = 4.1 × 10 −29 W;
1 LGacrux _
(b) d = _
8.32 × 10 −3
OR 120 pc; _ = 4.1 × 1026 = 1.1 × 10 3
29
(3)
L⊙ 3.85 × 10
120 × 3.26 × 3x 10 8 × 365x24 × 3600
(c) if the star is too far then the parallax angle is
= 3.70 × 10 18 m (2) too small to be measured OR stellar parallax is
Answer must be in meters, watch for POT. limited to closer stars (1)
(c) distances are so big/large OR to avoid using large
powers of 10 OR they are based on convenient
definitions (1)

40
Answers

(d) line or area roughly inside shape shown – judge (c) ​L = AT ​ ​​  4​= 5.67 × ​10​​  −8​ × 4 × ​​(2.0 × ​10​​  4​)​​​  2​ × ​​(​10​​  6​)​​​  4​​;
by eye ​L = 3 × ​10​​  26​ W OR L = 2.85 × ​10​​  26​  W​; (2)
Accept straight line or straight area at roughly 45° Allow ECF for [1 max.] if ​π​r used (gives 7 × 10 W)
2 26

10 6 (1) Allow ECF for a POT error in MP1.


​2.9 × ​10​​  ​ −3
105 λ=_
(d) ​ ​= 2.9 × ​10​​  −9​​ m;
6
​10​​  ​
104
this is an X-ray wavelength (2)
103
13 (a) A (1)
luminosity/Lo

102

10
(b) The binding energy per nucleon of uranium
is less than that of nuclei smaller than iron.
1
Therefore, energy would overall have been
10−1
transferred from the star to the uranium
10−2
nucleons.(1)
10−3

20 000 10 000 5000 2500


temperature/K

(e) at ​​10​​  3​ ​L⊙​  ​​​, further to right than 5000 K and to the
left of 2500 K (see shaded region)
106

105
(1)
104

103
luminosity/Lo

102

10

10−1

10−2
10−3

20 000 10 000 5000 2500


temperature/K

(f ) ALTERNATIVE 1
main sequence to red giant;
planetary nebula; with mass reduction/loss OR
planetary nebula; with mention of remnant mass;
white dwarf (3)
ALTERNATIVE 2
main sequence to red supergiant region;
supernova; with mass reduction/loss OR
supernova; with mention of remnant mass;
neutron star OR black hole (3)
12 (a) core: helium;
outer layer: hydrogen (2)
(b) line to the right of X, possibly undulating, very
roughly horizontal (1)
Ignore any paths beyond this as the star disappears from
diagram.

41

You might also like